New classes are starting soon - secure your spot today!

2017 AIME II Discussion

Go back to the Math Jam Archive

AoPS Instructors discuss all 15 problems on the 2017 AIME II .

Copyright © 2024 AoPS Incorporated. This page is copyrighted material. You can view and print this page for your own use, but you cannot share the contents of this file with others.

Facilitator: Dave Patrick

DPatrick 2017-03-24 19:00:26
Welcome to the 2017 AIME II Math Jam!
DPatrick 2017-03-24 19:00:33
I'm Dave Patrick, and I'll be leading our discussion tonight.
DPatrick 2017-03-24 19:00:42
Before we get started I would like to take a moment to explain our virtual classroom to those who have not previously participated in a Math Jam or one of our online classes.
DPatrick 2017-03-24 19:00:50
The classroom is moderated, meaning that students can type into the classroom, but these comments will not go directly into the room. These comments go to the instructors, who may choose to share your comments with the room.
This helps keep the class organized and on track. This also means that only well-written comments will be dropped into the classroom, so please take time writing responses that are complete and easy to read.
DPatrick 2017-03-24 19:01:07
Notice that this is a lot like one of our classes except there are a lot more of you and the same number of me. I won't be able to post all of your comments all of the time. It's not personal! Here's a secret though: I prefer to pass clear, well-written comments to the room.
DPatrick 2017-03-24 19:01:26
Also, we won't be going through the math quite as thoroughly as we do in our classes -- I can't teach all the necessary material for every problem as we go.
DPatrick 2017-03-24 19:01:37
Another difference between tonight and our regular online classes is that it is very unlikely that we'll be able to answer every single question you ask. We always to try do so in our regular online classes, but we have a large number of students tonight! So, please go ahead and ask questions, but also please understand if we aren't able to answer them all!
DPatrick 2017-03-24 19:01:48
We do have two teaching assistants with us tonight to help answer your questions.
DPatrick 2017-03-24 19:01:58
Jessica Oehrlein (numberdance): Jessie graduated from Olin College of Engineering with a degree in mechanical engineering, and she now studies applied math and atmospheric science at Columbia University. She has attended Canada/USA Mathcamp, RSI (Research Science Institute), and BSM (Budapest Semesters in Mathematics). In her free time, she contra dances, rides roller coasters, and watches far too much ballet and modern dance.
numberdance 2017-03-24 19:02:02
Hi y'all!
DPatrick 2017-03-24 19:02:26
Our second TA is unfortunately having some technical issues right now, but I'll introduce him anyway:
DPatrick 2017-03-24 19:02:31
Aakarsh Gottumukkala (numberman): Aakarsh majored in philosophy and minored in mathematics at The College of Wooster. Currently, he is a masters student in the liberal arts at St. John's College. He has been a Robinson Center Teaching Assistant for Mathematical Logic three summers in a row in Seattle, WA. He is really interested in algebra and number theory.
DPatrick 2017-03-24 19:02:45
Hopefully he'll be with us soon.
DPatrick 2017-03-24 19:02:59
Our TAs can answer questions by whispering to you or by opening a window with you to chat 1-on-1. However, due to the large size of the session tonight, they may not be able to get to you right away (or at all). Repeating your question over and over is more likely to annoy us than to get it answered faster, so please, just ask your question once and be patient, and please understand that we may not be able to answer all the questions tonight.
DPatrick 2017-03-24 19:03:23
Please also remember that the purpose of this Math Jam is to work through the solutions to AIME problems, and not to merely present the answers. "Working through the solutions" includes discussing problem-solving tactics. Also on occasion we may stop to prove things that you wouldn't necessary need to prove while doing the contest. So please, when a question is posted, do not simply respond with the final answer. That's not why we're here.
DPatrick 2017-03-24 19:03:56
Probably about halfway through, I will get thirsty or hungry, or my fingers will get tired, and I'll take a 1-2 minute break. Other than that, we will simply plow through all 15 problems. It'll take 2-3 hours.
DPatrick 2017-03-24 19:04:23
Let's get started!
DPatrick 2017-03-24 19:04:35
1. Find the number of subsets of $\{1,2,3,4,5,6,7,8\}$ that are subsets of neither $\{1,2,3,4,5\}$ nor $\{4,5,6,7,8\}$.
DPatrick 2017-03-24 19:04:55
(Note that I'll always pin the current problem at the top, so we can refer to it while we discuss the solution.)
lsh0589 2017-03-24 19:05:06
complementary counting?
SomethingNeutral 2017-03-24 19:05:06
complementary counting!!
Ani10 2017-03-24 19:05:06
They're literally telling you to do complementary counting
DPatrick 2017-03-24 19:05:38
Certainly one good option is to use complementary counting: count the subsets of the big set, and subtract the subsets of the smaller sets.
DPatrick 2017-03-24 19:05:56
For whatever reason, though, that didn't occur to me in real time -- I just did it directly.
alifenix- 2017-03-24 19:06:02
I first used constructive counting, then checked with complementary counting.
DPatrick 2017-03-24 19:06:21
Indeed: you can just construct a valid subset directly. (Complementary counting becomes a nice check.)
DPatrick 2017-03-24 19:06:32
What does it mean for a subset of the big set to not be a subset of $\{1,2,3,4,5\}$?
baseballcat 2017-03-24 19:06:51
it has to contain 6,7, or 8
antmath2520 2017-03-24 19:06:51
It must contain at least one number from ${6, 7, 8}$.
vishwathganesan 2017-03-24 19:06:54
it has at least one of 6, 7, or 8
62861 2017-03-24 19:06:54
it contains at least one element of {6, 7, 8}
DPatrick 2017-03-24 19:07:00
Right: it must have a 6, a 7, or an 8.
DPatrick 2017-03-24 19:07:07
And similarly, to not be a subset of $\{4,5,6,7,8\}$ it must have a 1, a 2, or a 3.
GeneralCobra19 2017-03-24 19:07:19
You can also do it another way, by using constructive... since you need to pick at least one from {1, 2, 3} and {6, 7, 8} and {4, 5} doesn't matter.
antmath2520 2017-03-24 19:07:22
It must contain a number from ${1, 2, 3}$ AND a number from ${6, 7, 8}$.
DPatrick 2017-03-24 19:07:37
Exactly. So we can construct a valid subset by looking at the different pieces.
DPatrick 2017-03-24 19:07:43
We can take any subset of $\{1,2,3\}$ except the empty set.



We can take any subset of $\{4,5\}$ (including the empty set).



We can take any subset of $\{6,7,8\}$ except the empty set.
DPatrick 2017-03-24 19:07:50
And then we mash them together.
DPatrick 2017-03-24 19:08:06
How many choices does that give us?
GeneralCobra19 2017-03-24 19:08:34
That makes it (2^3-1)(2^3-1)*2^2=196
SomethingNeutral 2017-03-24 19:08:34
so 7*4*7 = 196 !!! (not triple factorial)
arghh 2017-03-24 19:08:34
7 * 4 * 7 ways
willmathxu 2017-03-24 19:08:34
7*7*4
antmath2520 2017-03-24 19:08:34
$\left(2^3 - 1\right) \times 2^2 \times \left(2^3 - 1\right) = 7 \times 4 \times 7 = \boxed{196}$
baseballcat 2017-03-24 19:08:34
7*7*4
DPatrick 2017-03-24 19:08:40
There are $2^3 - 1 = 7$ choices of nonempty subset of $\{1,2,3\}$.



There are $2^2 = 4$ choices of any subset of $\{4,5\}$.



There are $2^3 - 1 = 7$ choices of nonempty subset of $\{6,7,8\}$.
DPatrick 2017-03-24 19:08:48
So, altogether, there are $7 \cdot 4 \cdot 7 = \boxed{196}$ possible subsets of the big set.
DPatrick 2017-03-24 19:09:11
2. Teams $T_1$, $T_2$, $T_3$, and $T_4$ are in the playoffs. In the semifinal matches, $T_1$ plays $T_4$, and $T_2$ plays $T_3$. The winners of those two matches will play each other in the final match to determine the champion. When $T_i$ plays $T_j$, the probability that $T_i$ wins is $\dfrac{i}{i+j}$, and the outcomes of all matches are independent. The probability that $T_4$ will be the champion is $\dfrac{p}{q}$, where $p$ and $q$ are relatively prime positive integers. Find $p+q$.
DPatrick 2017-03-24 19:09:33
What has to happen for $T_4$ to win it all?
SomethingNeutral 2017-03-24 19:10:05
t4 beats t1 and hwoever wins
baseballcat 2017-03-24 19:10:05
t4 beats t1 and beats the winner of t2 vs t3
GeronimoStilton 2017-03-24 19:10:05
First $T_4$ must beat $T_1$, and then $T_2$ or $T_3$
thedoge 2017-03-24 19:10:05
$T_4$ has to win the both games
popcorn1 2017-03-24 19:10:05
He beats T1 and then the winner of T2 and T3's match.
DPatrick 2017-03-24 19:10:07
First, $T_4$ must defeat $T_1$ in the semifinal. Then $T_4$ must defeat the winner of $T_2$-vs-$T_3$.
DPatrick 2017-03-24 19:10:18
What's the probability of $T_4$'s semifinal win?
mathrocks001 2017-03-24 19:10:38
4/5
phi_ftw1618 2017-03-24 19:10:38
$\frac{4}{5}$
mathmagician 2017-03-24 19:10:38
4/5
serd 2017-03-24 19:10:38
4/5
FieryEntei 2017-03-24 19:10:38
The probability of that happening is $\frac{4}{5}$ as given by the formula in the question
DPatrick 2017-03-24 19:10:43
We just use the formula given in the problem: it's $\dfrac{4}{4+1} = \dfrac45$.
DPatrick 2017-03-24 19:10:50
Now what?
carzland 2017-03-24 19:11:08
We could do casework based on who wins the semifinal match that $T_4$ is not in.
legolego 2017-03-24 19:11:08
casework whether T_2 or T_3 wins
vishwathganesan 2017-03-24 19:11:08
casework on who wins t2 vs t3?
DPatrick 2017-03-24 19:11:16
Right. We have two cases depending on whether $T_2$ or $T_3$ wins the other semifinal.
DPatrick 2017-03-24 19:11:28
Note that $T_2$ wins with probability $\dfrac{2}{2+3} = \dfrac25$ and $T_3$ wins with probability $\dfrac{3}{3+2} = \dfrac35$.
DPatrick 2017-03-24 19:11:34
If $T_2$ wins the semi, then what's the probability that $T_4$ defeats $T_2$?
noob_user 2017-03-24 19:11:52
2/3
MathTechFire 2017-03-24 19:11:52
2/3
goodball 2017-03-24 19:11:52
2/3
DPatrick 2017-03-24 19:11:56
It's $\dfrac{4}{4+2} = \dfrac46 = \dfrac23$.
DPatrick 2017-03-24 19:12:02
And if $T_3$ wins the semi, what's the probability that $T_4$ defeats $T_3$?
vy1234 2017-03-24 19:12:26
4/7
QuestForKnowledge 2017-03-24 19:12:26
4/7
vishwathganesan 2017-03-24 19:12:26
4/(3+4)=4/7
Ani10 2017-03-24 19:12:26
4/7
DPatrick 2017-03-24 19:12:29
It's $\dfrac{4}{4+3} = \dfrac47$.
DPatrick 2017-03-24 19:12:40
So how do we combine all this data? What's the overall probability of $T_4$ winning the championship?
Ani10 2017-03-24 19:12:52
so take a weighted average right?
DPatrick 2017-03-24 19:13:13
Right -- for the second game, we have to add the two cases, weighted by the probability of $T_2$ or $T_3$ winning.
tdeng 2017-03-24 19:13:19
(4/5)(2/3*2/5+4/7*3/5)
DPatrick 2017-03-24 19:13:30
Indeed: $T_4$ wins the semifinal with probability $\dfrac45$, then wins the final with probability $\dfrac25 \cdot \dfrac23 + \dfrac35 \cdot \dfrac47$.
DPatrick 2017-03-24 19:13:39
So the overall probability is \[\frac45 \cdot \left(\frac25 \cdot \frac23 + \frac35 \cdot \frac47\right).\]
mathrocks001 2017-03-24 19:13:55
256/525
GeronimoStilton 2017-03-24 19:13:55
256/625
thedoge 2017-03-24 19:13:55
so $\frac{256}{525}$
skiboy32 2017-03-24 19:13:55
so, the answer is $\frac{256}{525}$
DPatrick 2017-03-24 19:14:01
The common denominator inside the parentheses is $5 \cdot 3 \cdot 7 = 105$, so this is \[\frac45 \cdot \left(\frac{28}{105} + \frac{36}{105}\right) = \frac45 \cdot \frac{64}{105} = \frac{256}{525}.\]
DPatrick 2017-03-24 19:14:13
And this is already in lowest terms, so the final answer is $256 + 525 = \boxed{781}$.
DPatrick 2017-03-24 19:14:54
Don't get too hung up on the term "weighted average" -- that just means that when we combine the cases, each case doesn't have the same probability.
DPatrick 2017-03-24 19:15:04
We've "weighted" the cases by the probability that each one occurs.
DPatrick 2017-03-24 19:15:16
Anyway, onwards!
DPatrick 2017-03-24 19:15:20
3. A triangle has vertices $A(0,0)$, $B(12,0)$, and $C(8,10)$. The probability that a randomly chosen point inside the triangle is closer to vertex $B$ than to either vertex $A$ or vertex $C$ can be written as $\dfrac{p}{q}$, where $p$ and $q$ are relatively prime positive integers. Find $p+q$.
GeronimoStilton 2017-03-24 19:15:37
Diagram?
SomethingNeutral 2017-03-24 19:15:37
DRAw a diagram
QuestForKnowledge 2017-03-24 19:15:37
draw it
carzland 2017-03-24 19:15:37
Draw it!
serd 2017-03-24 19:15:37
graph it?
DPatrick 2017-03-24 19:15:41
Let's start with a picture -- notice that I'll draw it on "graph paper" like I hope you did in real time:
DPatrick 2017-03-24 19:15:44
DPatrick 2017-03-24 19:15:52
How do we compute the probability asked for in the problem?
mathman3880 2017-03-24 19:16:13
find the area
Skywalker64 2017-03-24 19:16:13
by area
legolego 2017-03-24 19:16:13
find desired region / total region
phi_ftw1618 2017-03-24 19:16:13
Find the set of points that are closest to B
reelmathematician 2017-03-24 19:16:13
area of favorable region
noob_user 2017-03-24 19:16:13
draw the feasible area
phi_ftw1618 2017-03-24 19:16:13
and then divide by area
DPatrick 2017-03-24 19:16:21
Right. We need to determine the region inside in the triangle that's closest to $B$, compute its area, and compare to the area of $\triangle ABC$.
DPatrick 2017-03-24 19:16:34
How do we determine the desired region?
dtxiong 2017-03-24 19:16:50
draw perpendicular bisectors
fdas 2017-03-24 19:16:50
Draw perpendicular bisectors
arghh 2017-03-24 19:16:50
Draw perpendicular bisectors of AB and BC
thedoge 2017-03-24 19:16:50
we draw the perpendicular bisectors of AB and BC
Funnybunny5246 2017-03-24 19:16:50
Perpendicular bisectors
reedmj 2017-03-24 19:16:50
First draw perpendicular bisectors of each side
spartan168 2017-03-24 19:16:50
Perpendicular bisectors
DPatrick 2017-03-24 19:16:58
Aha -- perpendicular bisectors!
DPatrick 2017-03-24 19:17:16
Let's start with "points closer to $B$ than to $A$".
goodball 2017-03-24 19:17:52
x>6
arghh 2017-03-24 19:17:52
That's the line x = 6, we want the region x > 6
Ani10 2017-03-24 19:17:52
perp bisector of AB
serd 2017-03-24 19:17:52
the line x=6
DPatrick 2017-03-24 19:18:01
Right: it must have $x$-coordinate 6 or more.
DPatrick 2017-03-24 19:18:05
So the blue region below is closer to $B$ than to $A$:
DPatrick 2017-03-24 19:18:12
DPatrick 2017-03-24 19:18:27
And now for points closer to $B$ than to $C$...?
reelmathematician 2017-03-24 19:18:43
perpendicular bisector of bc
QuestForKnowledge 2017-03-24 19:18:43
same thing
DPatrick 2017-03-24 19:19:00
Right. The points on the perpendicular bisector of $\overline{BC}$ are equally distant from $B$ and from $C$.
DPatrick 2017-03-24 19:19:18
So the points "below" that bisector in our diagram will be closer to $B$ than to $C$.
DPatrick 2017-03-24 19:19:22
DPatrick 2017-03-24 19:19:31
I've labeled the midpoint of $\overline{BC}$ as $Y$.
DPatrick 2017-03-24 19:19:39
How do we compute the area of $XBYZ$?
arghh 2017-03-24 19:20:02
Find the coordinates of Z
serd 2017-03-24 19:20:02
break it up into pieces or draw a big rectangle and subtract
DPatrick 2017-03-24 19:20:57
I'd probably find the coordinates of $Z$ and then just break it up into nice right triangles.
DPatrick 2017-03-24 19:21:08
How do we find point $Z$?
reelmathematician 2017-03-24 19:21:29
find the intersection of both bisectors
JJShan26 2017-03-24 19:21:29
set the two equations of the lines equal to each other
a1b2 2017-03-24 19:21:29
Set up a system of equations
serd 2017-03-24 19:21:29
system of equations of the perp. bisectors
mathrocks001 2017-03-24 19:21:33
slope of YZ
DPatrick 2017-03-24 19:21:57
Well, we know the $x$-coordinate of $Z$ is 6. So we just can find the equation for $\overline{YZ}$ and plug in $x=6$.
edward1000 2017-03-24 19:22:02
it's perpendicular to CB, and the slope of CB is -10/4, or -5/2, so the slope of ZY is 2/5
DPatrick 2017-03-24 19:22:16
Exactly. The slope of $\overline{BC}$ is $-\dfrac52$. So the slope of $\overline{YZ}$ is $\dfrac25$.
DPatrick 2017-03-24 19:22:33
Therefore, the equation of the line containing $Y$ and $Z$ is \[y-5 = \frac25(x-10).\]
rainbow10 2017-03-24 19:22:51
so y=2x/5+1
giacomorizzo 2017-03-24 19:22:51
y=(2/5)x + 1
DPatrick 2017-03-24 19:23:08
...or you could pretty easily write the slope-intercept form, since $(10,5)$ is an "easy" point.
reelmathematician 2017-03-24 19:23:13
plug in 6 for x
Ani10 2017-03-24 19:23:13
We know x=6, so plug in to find y
spartan168 2017-03-24 19:23:16
Next, plug in x = 6 to get y = 17/5 or (6, 17/5) for point Z
DPatrick 2017-03-24 19:23:21
$Z$ has $x$-coordinate 6, so when we plug $x=6$ into the equation for $\overline{YZ}$, we get \[y-5=\frac25(6-10).\]



This solves to get $y = \dfrac{17}{5}$.
DPatrick 2017-03-24 19:23:31
So $Z$ has coordinates $\left(6,\dfrac{17}{5}\right)$.
DPatrick 2017-03-24 19:23:55
And so now to compute the area of $XBYZ$...
DjokerNole 2017-03-24 19:24:07
rectangle - triangles
Ani10 2017-03-24 19:24:07
I would make a trapezoid and right triangle at this point by drawing x=10
reelmathematician 2017-03-24 19:24:07
break into 2 right triangles
mathrocks001 2017-03-24 19:24:16
create rectangle around 4 points
DPatrick 2017-03-24 19:24:19
All of these ideas are good.
DPatrick 2017-03-24 19:24:29
I did something even more basic:
DPatrick 2017-03-24 19:24:33
DPatrick 2017-03-24 19:24:46
The bottom piece $XPQZ$ is a rectangle with sides $\dfrac{17}{5}$ and 4, so its area is $\dfrac{68}{5}$.



The top piece $ZQY$ is a right triangle with legs 4 and $\dfrac85$, so its area is $\dfrac{16}{5}$.



The right piece $YBP$ is a right triangle with legs 5 and 2, so its area is 5.
DPatrick 2017-03-24 19:25:00
There's probably a dozen different good ways to compute the area of this quadrilateral.
DPatrick 2017-03-24 19:25:11
However you do it, the overall area of $XBYZ$ is $\dfrac{68}{5} + \dfrac{16}{5} + 5 = \dfrac{109}{5}$.
DPatrick 2017-03-24 19:25:28
And how do we finish?
DaAsianPotatoe 2017-03-24 19:25:41
divide by total area
legolego 2017-03-24 19:25:41
find area of whole triangle
GeronimoStilton 2017-03-24 19:25:41
Find the area of $ABC$
DPatrick 2017-03-24 19:25:46
Right. The original triangle $ABC$ has base $AB = 12$ and height 10, so its area is 60.
DPatrick 2017-03-24 19:25:56
Therefore, the probability that we want is \[\frac{[XBYZ]}{[ABC]} = \frac{\frac{109}{5}}{60} = \frac{109}{300}.\]
stardestroyer 2017-03-24 19:26:10
so 409
goodball 2017-03-24 19:26:10
Therefore, the answer is 409
DPatrick 2017-03-24 19:26:14
This cannot be further simplified, so our final answer is $109 + 300 = \boxed{409}$.
DPatrick 2017-03-24 19:26:34
This is a good argument for a "sanity check" of your answer at the end -- go re-read the problem and make sure you're answering what was asked for!
DPatrick 2017-03-24 19:26:54
Several of you went to $109 + 5 = 114$ here...but if you re-read the problem, there's no way 109/5 can be a probability!
DPatrick 2017-03-24 19:27:17
4. Find the number of positive integers less than or equal to 2017 whose base-three representation contains no digit equal to 0.
DPatrick 2017-03-24 19:27:27
What data would probably help?
arghh 2017-03-24 19:27:46
Figure out what 2017 is in base 3
serd 2017-03-24 19:27:46
write 2017 in base 3
to_chicken 2017-03-24 19:27:46
What $2017$ is in base $3$.
QuestForKnowledge 2017-03-24 19:27:46
what 2017 is in base 3
Fibonachos 2017-03-24 19:27:46
The base-3 representation of 2017
DPatrick 2017-03-24 19:28:00
We're probably going to need to find what 2017 is in base 3.
DPatrick 2017-03-24 19:28:06
You can easily work it out on your own, but to save time, here it is: \[ 2017 = 2 \cdot 3^6 + 2 \cdot 3^5 + 2 \cdot 3^3 + 2 \cdot 3^2 + 1 = 2202201_3.\]
DPatrick 2017-03-24 19:28:19
So what numbers in base 3 are less than this and don't contain a zero?
carzland 2017-03-24 19:28:51
Partial-Casework:
DPatrick 2017-03-24 19:29:04
Right, there are a couple of different cases to consider.
legolego 2017-03-24 19:29:07
have less than 7 digits or 7 digits and do not start with 22
DPatrick 2017-03-24 19:29:13
We can either have:

- a number with 6 digits or less, where each digit is 1 or 2; or

- a 7-digit number, where each digit is 1 or 2, provided it's less than $2202201_3$
DPatrick 2017-03-24 19:29:29
How many of the first type are there? (6 digits or less)
reaganchoi 2017-03-24 19:29:42
1 digit: 2

2 digit: 4

3 digit: 8

4 digit: 16

5 digit: 32

6 digit: 64
thedoge 2017-03-24 19:29:42
2^6+2^5+2^4+2^3+2^2+2^1
DPatrick 2017-03-24 19:29:48
For each digit, we have two choices: 1 or 2.
DPatrick 2017-03-24 19:29:54
So there are $2^1$ 1-digit numbers, $2^2$ 2-digit numbers, etc., up through $2^6$ 6-digit numbers.That is, there are $2^1 + 2^2 + \cdots + 2^6$ total.
Visconformity 2017-03-24 19:30:02
2^7-2
DPatrick 2017-03-24 19:30:15
Indeed, this is one less than $1 + 2 + \cdots + 2^6$, which itself is one less than $2^7$. So this quantity is $2^7 - 2 = 128 - 2 = 126$.
DPatrick 2017-03-24 19:30:33
How about of the second type? (7-digit numbers less than $2202201_3$)
Ani10 2017-03-24 19:31:00
i would separate into numbers that start with 11, 12, 21, and 22
GeronimoStilton 2017-03-24 19:31:07
They are $2122222_3$ or less.
JJShan26 2017-03-24 19:31:09
numbers that start with 21,11,12
DPatrick 2017-03-24 19:31:16
Good idea. The first two digits must be 11, 12, or 21. Then the last five digits can each either be 1 or 2.
mathrocks001 2017-03-24 19:31:32
32 for each
DPatrick 2017-03-24 19:31:37
This gives us 3 choices for the first two digits, then $2^5 = 32$ choices for the remaining five digits, for a total of $3 \cdot 32 = 96$ 7-digit numbers.
SomethingNeutral 2017-03-24 19:31:55
126+96=$\boxed{222}$
Archimedes15 2017-03-24 19:32:00
96 + the first 126, so our answer is 222
DPatrick 2017-03-24 19:32:01
So altogether, there are $126 + 96 = \boxed{222}$ such numbers.
DPatrick 2017-03-24 19:32:17
5. A set contains four numbers. The six pairwise sums of distinct elements of the set, in no particular order, are 189, 320, 287, 234, $x$, and $y$. Find the greatest possible value of $x+y$.
DPatrick 2017-03-24 19:32:39
Before doing anything else, let's organize our data first so that the four given numbers are in order:
DPatrick 2017-03-24 19:32:44
5. A set contains four numbers. The six pairwise sums of distinct elements of the set, in no particular order, are 189, 234, 287, 320, $x$, and $y$. Find the greatest possible value of $x+y$.
DPatrick 2017-03-24 19:32:54
And let's say our four numbers are $a < b < c < d$. (This is just to give us common ground to discuss the problem.)
DPatrick 2017-03-24 19:33:01
What other quantity might be helpful to know?
Archimedes15 2017-03-24 19:33:22
It might be helpful to know the sum of all of the numbers
skiboy32 2017-03-24 19:33:22
a+b+c+d
vishwathganesan 2017-03-24 19:33:22
their sum?
DPatrick 2017-03-24 19:33:37
The sum of all four numbers is probably key. Let's set $s = a + b + c + d$.
DPatrick 2017-03-24 19:33:43
But what else do we know about $s$?
edward1000 2017-03-24 19:33:57
3*s = the sum of all those numbers
SomethingNeutral 2017-03-24 19:34:00
189+234+287+320+x+y=3s
Littlelachy 2017-03-24 19:34:07
We know that 3s is 189+234+287+320+z+y
reelmathematician 2017-03-24 19:34:07
3 times the pariwise sum
shootingstar8 2017-03-24 19:34:09
189+234+287+320+x+y = 3s
DPatrick 2017-03-24 19:34:11
If we sum our six pairwise sums, we get $3s$, since each number in $\{a,b,c,d\}$ appears in three of the pairwise sums.
DPatrick 2017-03-24 19:34:20
So $189 + 234 + 287 + 320 + x + y = 3s$, and thus $x + y = 3s - 1030$.
DPatrick 2017-03-24 19:34:32
Thus, if we can maximize $s$, we'll automatically maximize $x+y$.
vishwathganesan 2017-03-24 19:34:43
s=(a+b)+(c+d)=(a+c)+(b+d)=(a+d)+(b+c)
DPatrick 2017-03-24 19:34:54
Aha, that's an good observation too.
DPatrick 2017-03-24 19:35:15
The pairwise sums themselves come in 3 pairs that each sum to $s$.
arghh 2017-03-24 19:35:29
It's got to be the sum of two of the given numbers
DPatrick 2017-03-24 19:35:40
Indeed, two of our four given sums must sum to $s$. That's because the pairwise sums come in three pairs of "opposites": $\{(a+b) \text{ and } (c+d)\}$, $\{(a+c) \text{ and } (b+d)\}$, and $\{(a+d) \text{ and } (b+c)\}$, so our four given sums must contain (at least) one of these pairs.
DPatrick 2017-03-24 19:35:49
So what?
a1b2 2017-03-24 19:36:03
pick two largest and let x and y solve itself
arghh 2017-03-24 19:36:06
Try 287 + 320, since we want maximum s?
rainbow10 2017-03-24 19:36:10
make 287 and 320 two of these pairs
liyuxiao 2017-03-24 19:36:10
maximize the two numbers to get the greatest amount for s
reelmathematician 2017-03-24 19:36:13
pick the 2 largest numbers
DPatrick 2017-03-24 19:36:16
Since we're trying to maximize $s$, we can hope that $s = 287 + 320 = 607$ is the sum of the two largest given numbers.
willmathxu 2017-03-24 19:36:40
1821-1030=x+y
DPatrick 2017-03-24 19:36:45
This would make $x+y = 3(607) - 1030 = 1821 - 1030 = 791$.
GeronimoStilton 2017-03-24 19:36:59
How do we know this works?
DPatrick 2017-03-24 19:37:04
Good question -- is this actually possible?
serd 2017-03-24 19:37:21
solve for a, b, c, and d
DPatrick 2017-03-24 19:37:35
Indeed it is: we can just try a configuration, such as $a+b = 189$, $a+c = 234$, $b+c = 287$, and $a+d = 320$.
vishwathganesan 2017-03-24 19:37:39
It says "distinct," so it may not work
DPatrick 2017-03-24 19:38:12
Right, that's really the only worry. They could have played a really nasty trick on us and given us a "maximal" solution that has two numbers that are equal.
GeneralCobra19 2017-03-24 19:38:23
You can just assign 189 to $a+b$ and onward...(234 to $a+c$, and onto the next two highest, which doesn't matter by the end) afterward, you can do some fun elimination(only three sums) to get desired b+c+2d.
DPatrick 2017-03-24 19:38:30
True. If you solve this (I won't take up our time doing so), you'll get $(a,b,c,d) = (68,121,166,252)$. (And for what it's worth, this gives $x = 121+252 = 373$ and $y = 166+252 = 418$ as the two missing sums, and indeed $373 + 418 = 791$, so that's a nice check.)
DPatrick 2017-03-24 19:38:49
So our answer is in fact $\boxed{791}$.
DPatrick 2017-03-24 19:39:00
And in fact, it turns out this is not the only configuration -- we can swap the last two sums and get $(a,b,c,d) = \{51.5, 137.5, 182.5, 235.5\}$ works too, giving $(x,y) = (373,418)$ again.
DPatrick 2017-03-24 19:39:25
One-third of the way through already!
DPatrick 2017-03-24 19:39:35
6. Find the sum of all positive integers $n$ such that $\sqrt{n^2 + 85n + 2017}$ is an integer.
DPatrick 2017-03-24 19:39:49
Gee, it'd be nice if we could write what's under the radical in terms of a square.
thedoge 2017-03-24 19:40:03
Complete the square
legolego 2017-03-24 19:40:03
complete the square
Funnybunny5246 2017-03-24 19:40:03
Complete square
DPatrick 2017-03-24 19:40:10
Good idea!
DPatrick 2017-03-24 19:40:22
But first, it's a slight pain to complete the square when the middle coefficient is odd, so what can we do?
62861 2017-03-24 19:40:44
multiply radicand by 4
SomethingNeutral 2017-03-24 19:40:44
Multiply by 4/4
thedoge 2017-03-24 19:40:44
multiply the inside by 4
goodball 2017-03-24 19:40:44
Multiply by 2
awesomemaths 2017-03-24 19:40:44
we could times 2
letsgomath 2017-03-24 19:40:49
multiply by 4
shark2018 2017-03-24 19:40:49
multiply by 2
avy 2017-03-24 19:40:49
multiply the whole square root by 2 and so the inside by 4
DPatrick 2017-03-24 19:40:54
I agree. Let's multiply under the radical by a 4, and put a $\dfrac12$ outside: \[ \sqrt{n^2 + 85n + 2017} = \frac12\sqrt{4n^2 + 4\cdot85n+ 8068}.\]
DPatrick 2017-03-24 19:41:16
So now when we complete the square, we get $\dfrac12\sqrt{(2n+85)^2+843}$.
DPatrick 2017-03-24 19:41:33
(Trust me -- the 843 is correct.)
DPatrick 2017-03-24 19:41:42
And if we want this to be an integer, let's give that integer a name: \[ \frac12\sqrt{(2n+85)^2 + 843} = m.\]
DPatrick 2017-03-24 19:41:46
Now what?
letsgomath 2017-03-24 19:42:14
843 is the difference of squares
reedmj 2017-03-24 19:42:14
Get rid of the radical
awesomemaths 2017-03-24 19:42:14
multiply 2 both sides
arghh 2017-03-24 19:42:14
Expand? (2n + 85)^2 + 843 = 4m^2
DPatrick 2017-03-24 19:42:27
Right, if we multiply by 2 and then square, we get $843 = 4m^2 - (2n+85)^2$.
DPatrick 2017-03-24 19:42:33
Aha, that's a difference of squares!
SomethingNeutral 2017-03-24 19:42:46
Difference of squares so factor 843 (3*281)
shootingstar8 2017-03-24 19:42:50
Find the factors of 843
DPatrick 2017-03-24 19:42:54
So it factors as $843 = (2m - 2n - 85)(2m + 2n + 85)$.
DPatrick 2017-03-24 19:43:04
Happily, the only ways we can factor $843$ into two positive integers are $843 = 1 \cdot 843 = 3 \cdot 281$ (note 281 is prime).
DPatrick 2017-03-24 19:43:19
How do we use this info most efficiently to complete the problem?
Ani10 2017-03-24 19:43:43
since n is positive, (phew) first factor is smaller than first, so only two cases
QuestForKnowledge 2017-03-24 19:43:43
second term must be bigger, solve system
DPatrick 2017-03-24 19:43:58
We could solve this completely, but we only care about $n$, right?
DPatrick 2017-03-24 19:44:14
How do we quickly isolate $n$?
fdas 2017-03-24 19:44:19
2n+85 is half of the difference
DPatrick 2017-03-24 19:44:34
Right! Or to say it another way: the difference in the two factors is $4n + 170$. This difference must equal $842$ or $278$.
DPatrick 2017-03-24 19:44:51
Solving $4n + 170 = 842$ gives $n = 168$.

Solving $4n + 170 = 278$ gives $n = 27$.
DPatrick 2017-03-24 19:45:18
To be fair, if we wanted to be complete, we should probably also check that $m$ is an integer. By summing the two factors we get $4m = 844$ or $4m = 284$, so $m=211$ or $m=71$ respectively.
celestialphoenix3768 2017-03-24 19:45:26
168+27=195
GeneralCobra19 2017-03-24 19:45:26
168+27 is 195
DPatrick 2017-03-24 19:45:35
And finally, the sum of the possible $n$'s is $168 + 27 = \boxed{195}$.
DPatrick 2017-03-24 19:46:03
This might have been my favorite problem -- it tickled me for some reason.
DPatrick 2017-03-24 19:46:17
7. Find the number of integer values of $k$ in the closed interval $[-500,500]$ for which the equation $\log(kx) = 2\log(x+2)$ has exactly one real solution.
arghh 2017-03-24 19:46:55
Get rid of the logs: kx = (x + 2)^2
SomethingNeutral 2017-03-24 19:46:55
expand logs into exponents
Archimedes15 2017-03-24 19:46:55
kx = (x+2)^2
DPatrick 2017-03-24 19:47:02
Right -- we can use the log exponent rule to rewrite the equation as $\log(kx) = \log\left((x+2)^2\right)$.
DPatrick 2017-03-24 19:47:09
And we know that two logs are equal if and only if the numbers themselves are equal.
DPatrick 2017-03-24 19:47:16
In other words, we have $kx = (x+2)^2$.
DPatrick 2017-03-24 19:47:32
So we're just looking for values of $k$ in $[-500,500]$ such that $kx = (x+2)^2$ has exactly one real solution...right?
arghh 2017-03-24 19:47:57
Though we need to make sure that kx > 0 and x + 2 > 0
gradysocool 2017-03-24 19:47:57
except no we must keep track of our original domains!!
DPatrick 2017-03-24 19:48:17
Right, this is the trick to this problem. Not all solutions of $kx = (x+2)^2$ are necessarily solutions to our original equation!
DPatrick 2017-03-24 19:48:26
We can only take logs of positive numbers.
DPatrick 2017-03-24 19:48:32
That is, we need $kx > 0$ and $x+2 > 0$ (or $x > -2$) in order to have a valid solution.
DPatrick 2017-03-24 19:48:49
So we can rewrite the question without logs if we put this condition in:
DPatrick 2017-03-24 19:48:53
7. Find the number of integer values of $k$ in the closed interval $[-500,500]$ for which the equation $kx = (x+2)^2$, with the restrictions $kx > 0$ and $x > -2$, has exactly one real solution.
DPatrick 2017-03-24 19:49:07
Now what?
fdas 2017-03-24 19:49:30
Move the kx on the other side and use quadratic formula
Funnybunny5246 2017-03-24 19:49:30
Expand square and move to one side
thedoge 2017-03-24 19:49:30
expand and use quadratic formula
DPatrick 2017-03-24 19:49:56
I like the "expand and move to one side" idea -- let's make this look like a "standard" quadratic -- but I'm not as fond of the quadratic formula here. Let's see...
DPatrick 2017-03-24 19:50:03
7. Find the number of integer values of $k$ in the closed interval $[-500,500]$ for which the equation $x^2 + (4-k)x + 4 = 0$, with the restrictions $kx > 0$ and $x > -2$, has exactly one real solution.
DPatrick 2017-03-24 19:50:18
(All I did was rearrange the quadratic.)
Fibonachos 2017-03-24 19:50:35
Look at the discriminant!!
letsgomath 2017-03-24 19:50:35
use the discriminant
DPatrick 2017-03-24 19:50:44
Sure, let's just look at the discriminant first.
DPatrick 2017-03-24 19:50:52
The discriminant of this quadratic is $(4-k)^2 - 16$.
DPatrick 2017-03-24 19:51:10
So there are no real roots if $0 < k < 8$, a double (real) root if $k=0$ or $k=8$, and two real roots if $k<0$ or $k>8$.
DPatrick 2017-03-24 19:51:19
We can't have $0 < k < 8$ -- there aren't any solutions at all.
vishwathganesan 2017-03-24 19:51:25
k=0 works, but it doesn't fit restrictions
fdas 2017-03-24 19:51:25
k cant equal 0
DPatrick 2017-03-24 19:51:34
We can't have $k=0$, because of the $kx > 0$ inequality.
pslv 2017-03-24 19:51:45
k=8
BooBooTM 2017-03-24 19:51:45
8 works, right?
DPatrick 2017-03-24 19:52:05
If $k=8$, then $x=2$ is a repeated root, and that's valid with our inequalities. So $k=8$ works.
DPatrick 2017-03-24 19:52:10
What happens if $k>8$?
Archimedes15 2017-03-24 19:52:32
two real roots... distinct
legolego 2017-03-24 19:52:32
two distinct valid real roots
skiboy32 2017-03-24 19:52:32
then, we get two positive roots, so nope
DPatrick 2017-03-24 19:52:53
We get two real roots, and they're both positive (since their sum is $k-4$ and their product is $4$).
DPatrick 2017-03-24 19:53:14
So these roots are both valid for our original problem. That's no good, because we only want one root.
DPatrick 2017-03-24 19:53:19
So none of $k>8$ work.
DPatrick 2017-03-24 19:53:23
What about $k<0$?
SomethingNeutral 2017-03-24 19:53:40
2 roots too
DPatrick 2017-03-24 19:53:57
Right: there are also 2 real roots of our quadratic. But what do we know about them?
DPatrick 2017-03-24 19:54:11
Remember, they sum to $k-4$ and their product is 4.
DPatrick 2017-03-24 19:55:06
We have two roots whose sum is negative but whose product is positive. What does that tell us?
serd 2017-03-24 19:55:18
theyre both negative
QuestForKnowledge 2017-03-24 19:55:18
2 negative roots
arghh 2017-03-24 19:55:21
They are both negative
shootingstar8 2017-03-24 19:55:21
The roots are negative?
DPatrick 2017-03-24 19:55:46
Right. If $k<0$, then the quadratic has two negative roots for $x$. That's good because that will make $kx > 0$ true.
DPatrick 2017-03-24 19:55:56
In which case will exactly one of these roots satisfy $x > -2$?
reelmathematician 2017-03-24 19:56:38
one root less than -2 and one more
DPatrick 2017-03-24 19:56:46
Right! If two different negative numbers multiply to give 4, then one must be smaller than $-2$ and one must be larger than $-2$.
DPatrick 2017-03-24 19:57:31
So when $k < 0$, our quadratic has two distinct negative roots: one less than $-2$ and one greater than $-2$. Only the second one satisfies $x > -2$, so only the second one is also a solution to our original problem.
vishwathganesan 2017-03-24 19:57:37
so all k<0 work?
DPatrick 2017-03-24 19:57:54
Exactly. All $k < 0$ will produce exactly one solution to our original problem.
flamescarlet 2017-03-24 19:58:14
so 501!
SomethingNeutral 2017-03-24 19:58:14
so answer $\boxed{501}?$
SimonSun 2017-03-24 19:58:14
so 500 negative values
GeronimoStilton 2017-03-24 19:58:14
$501$!
DPatrick 2017-03-24 19:58:16
Therefore, in the given interval, all $-500 \le k \le -1$ work, along with $k=8$.
DPatrick 2017-03-24 19:58:31
So the answer is $\boxed{501}$.
DPatrick 2017-03-24 19:59:19
8. Find the number of positive integers $n$ less than 2017 such that \[ 1 + n + \frac{n^2}{2!} + \frac{n^3}{3!} + \frac{n^4}{4!} + \frac{n^5}{5!} + \frac{n^6}{6!} \] is an integer.
GeronimoStilton 2017-03-24 19:59:42
This looks like $e^n$.
DPatrick 2017-03-24 20:00:03
Indeed, if you know some calculus, you'll recognize this as the first 8 terms of the Taylor series for $e^n$.
DPatrick 2017-03-24 20:00:16
You can completely ignore that last sentence though, as it is of absolutely no help in solving the problem.
DPatrick 2017-03-24 20:00:27
(And I guess it's only the first 7 terms.)
Fibonachos 2017-03-24 20:00:33
We don't need to worry about $1+n$
DPatrick 2017-03-24 20:00:43
That's a start: clearly we don't have to worry about the $1+n$ part of the expression. So let's simplify the problem.
DPatrick 2017-03-24 20:00:47
8. Find the number of positive integers $n$ less than 2017 such that \[ \frac{n^2}{2!} + \frac{n^3}{3!} + \frac{n^4}{4!} + \frac{n^5}{5!} + \frac{n^6}{6!} \] is an integer.
flamescarlet 2017-03-24 20:01:00
we can make 6! common denominator
hunter117 2017-03-24 20:01:11
we can factor out $n^2$
DPatrick 2017-03-24 20:01:12
Sounds good. We can factor out $n^2$, and put the rest of the terms over a common denominator:
DPatrick 2017-03-24 20:01:16
8. Find the number of positive integers $n$ less than 2017 such that \[ \frac{n^2(360 + 120n + 30n^2 + 6n^3 + n^4)}{720} \] is an integer.
Ani10 2017-03-24 20:01:27
prime factorize denominators?
giacomorizzo 2017-03-24 20:01:27
does n must have 2, 3, and 5 as factors
skiboy32 2017-03-24 20:01:27
it can be a multiple of $2\cdot 3\cdot 5=30$
DPatrick 2017-03-24 20:01:35
Right. We have to worry that the denominator "cancels out" with the numerator to leave an integer.
DPatrick 2017-03-24 20:01:40
So prime factors of these numbers might be more helpful.
DPatrick 2017-03-24 20:01:44
8. Find the number of positive integers $n$ less than 2017 such that \[ \frac{n^2\left((2^3\cdot3^2\cdot 5) + (2^3 \cdot 3 \cdot 5)n + (2 \cdot 3 \cdot 5)n^2 + (2 \cdot 3)n^3 + n^4)\right)}{2^4 \cdot 3^2 \cdot 5} \] is an integer.
DPatrick 2017-03-24 20:01:59
So we've got to worry about 2's, 3's, and 5's.
DPatrick 2017-03-24 20:02:11
And the nice thing is: we can worry about them separately!
DPatrick 2017-03-24 20:02:23
What's the condition on $n$ that will cancel the $2^4$ in the denominator?
legolego 2017-03-24 20:02:49
it is even
fdas 2017-03-24 20:02:49
multiple of 2
shootingstar8 2017-03-24 20:02:49
Having 2 as a factor
skiboy32 2017-03-24 20:02:49
$n$ is divisible by $2.$
hunter117 2017-03-24 20:02:49
$n$ must have a factor of $2^1$
DPatrick 2017-03-24 20:03:16
Certainly if $n$ is odd, then the numerator is odd ($n^2$) times odd (every term in the big parentheses is even except for $n^4$, which is odd), so the numerator overall is odd. We don't even cancel a single 2, much less $2^4$!
DPatrick 2017-03-24 20:03:31
If $n$ is merely even, do we have enough powers of 2 in the numerator?
legolego 2017-03-24 20:03:48
yes
Funnybunny5246 2017-03-24 20:03:48
Yes
avy 2017-03-24 20:03:48
yes
DPatrick 2017-03-24 20:03:57
Yes, way more than enough: we get at least a $2^2$ from the outer $n^2$, and each term inside the parentheses has at least $2^3$.
DPatrick 2017-03-24 20:04:12
So we cancel the 2's if and only if $n$ is even.
DPatrick 2017-03-24 20:04:17
How about the 3's? What's the condition on $n$ to cancel $3^2$?
carzland 2017-03-24 20:04:40
$n$ must be divisible by 3.
FieryEntei 2017-03-24 20:04:40
multiples of 3?
SimonSun 2017-03-24 20:04:40
Similar logic about 3, n is divisible by 3
colinyao 2017-03-24 20:04:40
n is divisible by 3
SomethingNeutral 2017-03-24 20:04:40
3|n
vishwathganesan 2017-03-24 20:04:40
div. by 3
DPatrick 2017-03-24 20:04:49
If $n$ is not a multiple of 3, then we're multiplying two non-multiples of 3 in the numerator, and we don't even cancel a single 3.
DPatrick 2017-03-24 20:05:05
If $n$ is a multiple of 3, then we get our $3^2$ just from the outer $n^2$ term, and don't need to worry about the 3's inside the big parentheses.
DPatrick 2017-03-24 20:05:13
So we cancel the 3's if and only if $n$ is a multiple of 3.
DPatrick 2017-03-24 20:05:18
How about the 5's? What's the condition on $n$ to cancel the 5 in the denominator?
flamescarlet 2017-03-24 20:05:46
n is a multiple of 5 or 6n^3 + n^4 is a multiple of 5
bertasaurus 2017-03-24 20:05:51
n must be divisible by 5 or (n+1) must be divisible by 5
DPatrick 2017-03-24 20:05:59
Right. First, if $n$ is a multiple of 5, we're set, because everything in sight is then a multiple of 5.
DPatrick 2017-03-24 20:06:24
Then, if $n$ is not a multiple of 5, we need $6n^3 + n^4$ to be a multiple of 5, because the other terms are already multiples of 5.
DPatrick 2017-03-24 20:06:34
And this is a multiple of 5 if and only if $1 + n$ is a multiple of 5 (we can factor out $n^3$, and 6 is the same as 1 modulo 5).
DPatrick 2017-03-24 20:06:53
So we cancel the 5's if and only if $n$ is either a multiple of 5 or one less than a multiple of 5.
DPatrick 2017-03-24 20:07:03
How do we combine all this info to count the $n$ that work?
hunter117 2017-03-24 20:07:17
chinese remainder theorem
flamescarlet 2017-03-24 20:07:17
chinese remainder theorem!
letsgomath 2017-03-24 20:07:20
has to satisfy all three mod congruences
DPatrick 2017-03-24 20:07:24
By the Chinese Remainder Theorem, we can combine our facts about 2's, 3's, and 5's into a single fact about what $n$ looks like in relationship to multiples of 30.
FTW 2017-03-24 20:07:46
0 or 24 mod 30
legolego 2017-03-24 20:07:46
equivalent to 0 or 24 mod 30
DPatrick 2017-03-24 20:08:04
Specifically, we need $n$ to be a multiple of 30 or 6 less than a multiple of 30. Or, to say it another way, we must have $n$ equivalent to 0 or 24 mod 30.
Mathguy1492 2017-03-24 20:08:14
$30n$ or $30n-6$
DPatrick 2017-03-24 20:08:21
Right.
DPatrick 2017-03-24 20:08:30
For every block of 30 integers, 2 of them will work.
Fibonachos 2017-03-24 20:08:36
There are 67 multiples of 30
DPatrick 2017-03-24 20:08:43
Yes -- since $2017 = 67 \cdot 30 + 7$, we have 67 full blocks of 30, plus 7 extra.
DPatrick 2017-03-24 20:09:31
Actually only 6 extra since we want "less than 2017" from the problem statement.
DPatrick 2017-03-24 20:09:57
But the "extra" are 2011, 2012, ..., 2016, and none of those work. (These are 1,2,...,6 mod 30, and the only numbers that work are either 0 or 24 mod 30.)
shark2018 2017-03-24 20:10:07
67*2= 134
flamescarlet 2017-03-24 20:10:07
so 67 * 2 = 134
arghh 2017-03-24 20:10:07
67 * 2
DPatrick 2017-03-24 20:10:19
Each of the 67 blocks of 30 have two numbers that work. So there are $2 \cdot 67 = \boxed{134}$ values of $n$ that work.
DPatrick 2017-03-24 20:10:40
9. A special deck of cards contains 49 cards, each labeled with a number from 1 to 7 and colored with one of seven colors. Each number-color combination appears on exactly one card. Sharon will select a set of eight cards from the deck at random. Given that she gets at least one card of each color and at least one card with each number, the probability that Sharon can discard one of her cards and still have at least one card of each color and at least one card with each number is $\dfrac{p}{q}$, where $p$ and $q$ are relatively prime positive integers. Find $p+q$.
DPatrick 2017-03-24 20:11:03
Let's give these situations names, so we can more easily discuss them.
DPatrick 2017-03-24 20:11:08
Let's say a "good" hand is a hand of 8 cards which contains all 7 numbers and all 7 colors.

Let's say a "great" hand is a good hand that has a discard that still leaves all 7 numbers and all 7 colors.
DPatrick 2017-03-24 20:11:15
So we want the probability that a good hand is also great.
DPatrick 2017-03-24 20:11:29
Any observations we can make before we start counting?
Mudkipswims42 2017-03-24 20:11:54
One duplicate
GeronimoStilton 2017-03-24 20:12:01
Well, by the Pigeonhole Principle, she has 2 of one color, and 2 of one number.
DPatrick 2017-03-24 20:12:14
Right: in any good hand, there are 2 cards with the same color, and 2 cards with the same number.
DPatrick 2017-03-24 20:12:32
And what makes a great hand great?
ESWN123456789 2017-03-24 20:12:50
If the hand is great, the double number and double color overlap
vishwathganesan 2017-03-24 20:12:50
in a great hand, the "bonus" color/number are the same card
DPatrick 2017-03-24 20:13:09
Right! We know every good hand must have exactly 2 cards with the same number (let's call that the "special number"), and exactly 2 cards with the same color (the "special color").
DPatrick 2017-03-24 20:13:26
When the special number and the special color appear on the same card, then we can discard that special card and still be left will all 7 numbers and all 7 colors. So that's what makes a good hand great.
DPatrick 2017-03-24 20:13:46
The hands that are good but not great have the special number on non-special colors and the special color on non-special numbers.
DPatrick 2017-03-24 20:13:57
With these observations in place, let's try to count!
DPatrick 2017-03-24 20:14:01
How many great hands are there?
JJShan26 2017-03-24 20:14:10
there are 7! ways to have all 7 numbers and 7 colors on 7 cards
DPatrick 2017-03-24 20:14:29
Right. We could first choose the 7 cards that remain after the discard. These have to have 7 numbers and 7 colors on 7 cards.
DPatrick 2017-03-24 20:14:50
We can construct such a 7-card set.



We need to choose one of the 7 colors for the 1.



Then we need to choose one of the 6 remaining colors for the 2.



Then we need to choose one of the 5 remaining colors for the 3.



And so on.
DPatrick 2017-03-24 20:15:01
So there are $7!$ ways to choose a perfect 7-card set.
DPatrick 2017-03-24 20:15:09
And then how many ways to choose an 8th card to make a great hand?
letsgomath 2017-03-24 20:15:25
42
legolego 2017-03-24 20:15:25
42
DPatrick 2017-03-24 20:15:40
Right. We can choose any card left in the deck! That card will be the "special card" in our 8-card great hand.
DPatrick 2017-03-24 20:15:48
Since there are 42 cards left, there are $7! \cdot 42$ ways to make a great hand.
DPatrick 2017-03-24 20:16:02
A couple people are asking if we double-counted when we did this...did we?
SimonSun 2017-03-24 20:16:28
no
Metal_Bender19 2017-03-24 20:16:28
no
DPatrick 2017-03-24 20:16:59
I don't think we did. If we have a great hand, then the discarded card is uniquely determined, and the 7-card set it leaves behind is also uniquely determined.
DPatrick 2017-03-24 20:17:17
(If we discarded any other card, we'll be losing a non-special number or color.)
DPatrick 2017-03-24 20:17:27
So there are $7! \cdot 42$ great hands.
DPatrick 2017-03-24 20:17:35
Next, how many "good but not great" hands are there?
DPatrick 2017-03-24 20:17:46
Recall: the hands that are good but not great have the special number on non-special colors and the special color on non-special numbers.
DPatrick 2017-03-24 20:18:03
How can we construct such a hand?
DPatrick 2017-03-24 20:18:56
Generally, when we constructively count, we try to count the most restrictive part of the construction first.
DPatrick 2017-03-24 20:19:22
Here the restriction is that we need two cards with the same number, and we need two cards with the same color, but these four cards are all different.
DPatrick 2017-03-24 20:19:49
There are probably different ways you could do it, but here's what I suggest:
DPatrick 2017-03-24 20:19:57
We have 7 choices for the special number.
DPatrick 2017-03-24 20:20:03
Then, $\dbinom72$ choices for the two colors that get paired with the special number.
DPatrick 2017-03-24 20:20:15
Then, 5 choices for the special color (we can't use either of the two colors from above, or else we'll end up making a special card, and we'll get a great hand).
DPatrick 2017-03-24 20:20:26
Then, $\dbinom62$ choices for the two numbers that get paired with the special color (we can't use the special number, for the same reason).
DPatrick 2017-03-24 20:20:40
This leaves 4 more cards, 4 unused numbers, and 4 unused colors. How many ways to finish the hand from here?
letsgomath 2017-03-24 20:20:58
24
Metal_Bender19 2017-03-24 20:20:58
4!
GeronimoStilton 2017-03-24 20:20:58
We have $24$ choices for that.
shark2018 2017-03-24 20:20:58
4!
Mathguy1492 2017-03-24 20:20:58
24
DPatrick 2017-03-24 20:21:20
We can pick these in $4!$ ways (we have to match up numbers with colors, like we did with our 7-card set from before).
DPatrick 2017-03-24 20:21:29
So there are a total of \[7 \cdot \binom72 \cdot 5 \cdot \binom62 \cdot 4!\] ways to make a good-not-great hand.
DPatrick 2017-03-24 20:21:57
If we write this out, this is \[\frac{7 \cdot 7 \cdot 6 \cdot 5 \cdot 6 \cdot 5 \cdot 4!}{4},\] so if we pull out a factor of $7!$ (to make it look more like our "great" count), we get $7! \cdot \dfrac{7 \cdot 6 \cdot 5}{4} = 7! \cdot \dfrac{105}{2}$.
DPatrick 2017-03-24 20:22:20
And how do we finish the problem from here?
vishwathganesan 2017-03-24 20:22:46
find probability
DPatrick 2017-03-24 20:22:55
We want \[P = \frac{\text{great}}{\text{good}} = \frac{\text{great}}{\text{great}+\text{good-not-great}}.\]
DPatrick 2017-03-24 20:23:10
Plugging in our counts, this is \[P = \frac{7! \cdot 42}{7! \cdot 42 + 7! \cdot \frac{105}{2}}.\]
letsgomath 2017-03-24 20:23:23
factor out 7!
flamescarlet 2017-03-24 20:23:23
factor out 7!
DPatrick 2017-03-24 20:23:33
The $7!$'s cancel, and multiplying through by 2 we're left with $\dfrac{84}{84 + 105}$.
Mathguy1492 2017-03-24 20:23:42
Then factor out another 7
letsgomath 2017-03-24 20:23:52
21
DPatrick 2017-03-24 20:23:54
Indeed, there's a common factor of 21 here.
DPatrick 2017-03-24 20:24:05
This is just \[\frac{84}{84+105}=\frac4{4+5}=\frac49.\]
DPatrick 2017-03-24 20:24:15
So the final answer is $4+9 = \boxed{013}$.
DPatrick 2017-03-24 20:24:47
Someone asked me why we didn't just could all the "good" hands. I didn't find a good way to do that except by casework into "great" and "good-not-great".
DPatrick 2017-03-24 20:25:20
Now's a good time for a short break, before we get to the double-digit problems!
DPatrick 2017-03-24 20:25:24
Let's resume at :30 past.
DPatrick 2017-03-24 20:30:09
And we're back!
DPatrick 2017-03-24 20:30:13
10. Rectangle $ABCD$ has side lengths $AB = 84$ and $AD = 42$. Point $M$ is the midpoint of $\overline{AD}$, point $N$ is the trisection point of $\overline{AB}$ closer to $A$, and point $O$ is the intersection of $\overline{CM}$ and $\overline{DN}$. Point $P$ lies on the quadrilateral $BCON$, and $\overline{BP}$ bisects the area of $BCON$. Find the area of $\triangle CDP$.
DPatrick 2017-03-24 20:30:26
Mmmmmm......bacon.....
GeronimoStilton 2017-03-24 20:30:44
Diagram?
shootingstar8 2017-03-24 20:30:44
Diagram!
awesomemaths 2017-03-24 20:30:44
diagram
DPatrick 2017-03-24 20:30:53
Sure, let's draw a picture. It's also pretty easy to add coordinates in case we want them:
DPatrick 2017-03-24 20:30:57
GeneralCobra19 2017-03-24 20:31:07
Can we coordinate bash this one?
DPatrick 2017-03-24 20:31:20
That seems promising to me. What are the coordinates of point $O$?
legolego 2017-03-24 20:31:55
(12, 24)
Skywalker64 2017-03-24 20:31:55
(12,24)
DPatrick 2017-03-24 20:32:02
Line $CM$ has slope $\dfrac14$ and $y$-intercept 21, so its equation is $y = \dfrac14x + 21$.



Line $DN$ has slope $-\dfrac32$ and $y$-intercept 42, so its equation is $y = -\dfrac32x + 42$.
DPatrick 2017-03-24 20:32:14
Equating these and solving gives $O = (12,24)$.
DPatrick 2017-03-24 20:32:30
(You can work it out on your own if you don't believe me.)
DPatrick 2017-03-24 20:32:41
Now what?
jonzli123 2017-03-24 20:32:53
find area of BCON
carzland 2017-03-24 20:32:53
Find the area.
shootingstar8 2017-03-24 20:32:56
Find point P
DPatrick 2017-03-24 20:33:09
We need to figure out where $P$ is. So that probably means finding the area $[BCON]$.
DPatrick 2017-03-24 20:33:19
We can find the area of $BCON$ by drawing in $\overline{BO}$ and computing the areas of the two triangles $BON$ and $BOC$.
DPatrick 2017-03-24 20:33:21
DPatrick 2017-03-24 20:33:50
This is pretty straightforward since we can just read the bases and the heights right from the picture.
DPatrick 2017-03-24 20:34:09
We have $[BON] = \dfrac12 \cdot 24 \cdot 56 = 672$ and $[BOC] = \dfrac12 \cdot 42 \cdot 72 = 1512$.
DPatrick 2017-03-24 20:34:28
(None of these computations are particularly interesting so I'm just going to skip the steps in the interest of time.)
GeronimoStilton 2017-03-24 20:34:34
$2184$
DPatrick 2017-03-24 20:34:38
Therefore, $[BCON] = 672 + 1512 = 2184$, and thus $\overline{BP}$ splits it into two regions of area $1092$.
DPatrick 2017-03-24 20:34:47
More specifically, where is $P$?
Ani10 2017-03-24 20:35:07
on OC
jonzli123 2017-03-24 20:35:07
on OC
GeronimoStilton 2017-03-24 20:35:07
On line $CO$
DPatrick 2017-03-24 20:35:22
Right. It lies on $\overline{OC}$, and it's far enough down so that $[BOP] = 1512 - 1092 = 420$.
DPatrick 2017-03-24 20:35:28
DPatrick 2017-03-24 20:36:00
Is there a good way to describe how far down $P$ is on $\overline{OC}$?
DPatrick 2017-03-24 20:36:35
What I mean is...sure we could find the coordinates of $P$, but do we really need to?
carzland 2017-03-24 20:37:44
We just need the length of the base.
DPatrick 2017-03-24 20:38:00
In fact, $[OBC]$ and $[PBC]$ share the same base $BC$, right?
DPatrick 2017-03-24 20:38:26
So $\dfrac{PC}{OC} = \dfrac{[PBC]}{[OBC]}$.
DPatrick 2017-03-24 20:38:58
And we already knew $[PBC] = 1092$ and $[OBC] = 1512$, so $\dfrac{PC}{OC} = \dfrac{1092}{1512}$.
DPatrick 2017-03-24 20:39:19
This simplifies to $\dfrac{PC}{OC} = \dfrac{13}{18}$.
DPatrick 2017-03-24 20:39:32
And what does that tell us about $[CDP]$?
GeronimoStilton 2017-03-24 20:40:14
It's $13 \cdot 42$
shootingstar8 2017-03-24 20:40:14
18 * CDP = 13 * OBC
fdas 2017-03-24 20:40:14
It has 13/18 the height of COD
DPatrick 2017-03-24 20:40:37
Right! We don't need to compute the coordinates of $P$, since we know that $\dfrac{[CDP]}{[CDO]} = \dfrac{PC}{OC}$ (since they share the base $CD$).
DPatrick 2017-03-24 20:40:56
So we have that $[CDP] = \dfrac{13}{18}[CDO]$.
_--__--_ 2017-03-24 20:41:15
84 * 13 / 2 = 546
GeronimoStilton 2017-03-24 20:41:22
$546$
DPatrick 2017-03-24 20:41:27
This gives $[CDP] = \dfrac{13}{18}\left(\dfrac12 \cdot 18 \cdot 84\right) = 13 \cdot 42 = \boxed{546}$.
DPatrick 2017-03-24 20:41:58
It would have been perfectly fine to find the coordinates of $P$, but I always like using common bases and/or common heights when we have them.
DPatrick 2017-03-24 20:42:09
11. Five towns are connected by a system of roads. There is exactly one road connecting each pair of towns. Find the number of ways there are to make all the roads one-way in such as way that it is still possible to get from any town to any other town using the roads (possibly passing through other towns on the way).
DPatrick 2017-03-24 20:42:49
Or... we can ask the opposite: when is it not possible to travel between any two towns?
a1b2 2017-03-24 20:43:09
They can't all point to/from a town
DPatrick 2017-03-24 20:43:25
Right. If there's a town where all the roads leave town and none arrive into town, then there's no way to travel to that town. Such a town is called a source.
DPatrick 2017-03-24 20:43:38
Similarly, if there's a town where all the roads arrive into town and none leave town, then there's no way to travel away from that town. Such a town is called a sink.
DPatrick 2017-03-24 20:43:50
These clearly fail.
DPatrick 2017-03-24 20:43:57
Are those the only situations where the road system fails? That is, if we know that no town is a source or a sink, can we conclude that we can travel from any town to any other town?
a1b2 2017-03-24 20:44:31
All others work.
DPatrick 2017-03-24 20:44:35
Can we prove this?
GeneralCobra19 2017-03-24 20:45:02
intuition?
palatine 2017-03-24 20:45:02
yes, probably
DPatrick 2017-03-24 20:45:33
The proof has a couple of subtle points, and in contest conditions I might be tempted to believe "it's obvious" and move on with the counting.
DPatrick 2017-03-24 20:45:47
But let's quickly glance at the proof.
DPatrick 2017-03-24 20:45:59
Specifically, suppose no town is a source or a sink. We'll prove that we can get from any town to any other town.
to_chicken 2017-03-24 20:46:15
There will always be a 'triangle' of towns that you can go to and from.
DPatrick 2017-03-24 20:46:25
Indeed, one good method is to find the largest cycle that you can: that is, a sequence of roads that form a loop between 3 or more towns.
DPatrick 2017-03-24 20:46:34
Obviously if you can find a cycle that includes all 5 towns, we're done, because we can just travel around the loop to get from anywhere to anywhere.
DPatrick 2017-03-24 20:46:45
Suppose you can find a cycle between 4 of the towns:
DPatrick 2017-03-24 20:46:48
DPatrick 2017-03-24 20:47:13
Can we always get to and from the 5th town?
vishwathganesan 2017-03-24 20:47:25
yes
anshrai 2017-03-24 20:47:25
yes
a1b2 2017-03-24 20:47:27
Yes because it is neither source nor sink.
tdeng 2017-03-24 20:47:32
Yes, because there is no sink or source
DPatrick 2017-03-24 20:47:35
Right, that's no problem either, because one of the 4 towns on the cycle will have a road to the 5th town, and one of the 4 towns on the cycle will have a road from the 5th town. (Otherwise the 5th town would be a source or a sink.)
DPatrick 2017-03-24 20:47:50
So we can get to the town off the cycle from the cycle, and vice versa, and hence all 5 towns are connected to each other.
DPatrick 2017-03-24 20:47:56
Finally, suppose you can only find a cycle with 3 towns:
DPatrick 2017-03-24 20:47:59
DPatrick 2017-03-24 20:48:11
I've also drawn the road between the other two towns (called X and Y); it doesn't matter which way it goes.
Mudkipswims42 2017-03-24 20:48:38
Y has to travel to at least one of the other towns, and have arrivals from at least one
reaganchoi 2017-03-24 20:48:41
Y must lead into the loop; X must have an entrance from the loop.
DPatrick 2017-03-24 20:48:43
Since Y isn't a sink, we have to be able to get from Y to the cycle.
DPatrick 2017-03-24 20:48:46
And since X isn't a source, we have to be able to get from the cycle to X.
DPatrick 2017-03-24 20:48:49
DPatrick 2017-03-24 20:49:07
We don't know which town(s) the dotted roads connect to in the cycle, but it doesn't matter!
DPatrick 2017-03-24 20:49:24
So we can get from any town to any town.
DPatrick 2017-03-24 20:49:46
By the way, before we proceed with the counting: does this still work if there are more than 5 towns?
fdas 2017-03-24 20:50:02
No. 6 towns can form 2 loops
ESWN123456789 2017-03-24 20:50:17
No with 6 there could be two cycles of 3
DPatrick 2017-03-24 20:50:42
Exactly. If we had 6 towns, then we could have a loop between 3 of them (call them A,B,C) and a different loop between the other 3 (call them X,Y,Z).
DPatrick 2017-03-24 20:51:19
And then, if all the roads went from {A,B,C} to {X,Y,Z}, there'd be no way to get from the second loop back to the first. (For example, no way to travel from Y to C.)
DPatrick 2017-03-24 20:51:37
So it's not quite so simple for 6 or more towns!
DPatrick 2017-03-24 20:51:50
Happily, we only have 5 towns, so now we know that our answer is \[(\text{total number of road arrangements}) - (\text{arrangements with a source or a sink}).\]
DPatrick 2017-03-24 20:51:58
How do we count this?
Mudkipswims42 2017-03-24 20:52:12
PIE
GeronimoStilton 2017-03-24 20:52:12
PIE?
RedPhoenix 2017-03-24 20:52:17
use PIE for the source or sink, subtract from 1024
DPatrick 2017-03-24 20:52:52
Right: for the second term, we can count "source arrangements" and "sink arrangements" separately, but that will double-count arrangements that have both.
DPatrick 2017-03-24 20:53:01
So to refine our answer a bit, using PIE, it's \[(\text{total number}) - (\text{with a source}) - (\text{with a sink}) + (\text{with both}).\]
DPatrick 2017-03-24 20:53:13
What's the total number of possible road arrangements?
to_chicken 2017-03-24 20:53:35
Total number of road arrangements is $2^{\binom{5}{2}}=1024$.
Diophantine33 2017-03-24 20:53:35
1024
DPatrick 2017-03-24 20:53:41
There are $\dbinom52 = 10$ roads, and each can go in either of 2 directions. So there are $2^{10} = 1024$ total road arrangements.
DPatrick 2017-03-24 20:53:48
How many with a source?
Metal_Bender19 2017-03-24 20:54:23
320
GeronimoStilton 2017-03-24 20:54:23
$5 \cdot 64$
jonzli123 2017-03-24 20:54:23
320
DPatrick 2017-03-24 20:54:28
We have 5 choices for the town that's the source (note that we can't have more than one!), and that fixes those 4 roads to go out of that town.
DPatrick 2017-03-24 20:54:36
The other 6 roads still have 2 choices each, so that $5 \cdot 2^6 = 5 \cdot 64 = 320$ arrangements with a source.
DPatrick 2017-03-24 20:54:44
How many with a sink?
to_chicken 2017-03-24 20:54:58
same: 320
Mudkipswims42 2017-03-24 20:54:58
same
tdeng 2017-03-24 20:54:58
Same number.
colinyao 2017-03-24 20:54:58
same as with a source
DPatrick 2017-03-24 20:55:05
By symmetry, it's the same: 320. (Or you could do the same computation.)
DPatrick 2017-03-24 20:55:10
How many with both?
jonzli123 2017-03-24 20:55:38
20*8
Metal_Bender19 2017-03-24 20:55:38
160
SomethingNeutral 2017-03-24 20:55:40
5*4*8 = 160
DPatrick 2017-03-24 20:55:44
We have 5 choices for the source and then 4 choices for the sink. This fixes 7 of the roads, leaving the other 3 (the roads running between the 3 remaining towns) free to choose their direction. So that's $5 \cdot 4 \cdot 2^3 = 20 \cdot 8 = 160$ with both.
soojoong 2017-03-24 20:55:59
$544$!!
DPatrick 2017-03-24 20:56:00
Therefore, our final answer is $1024 - 320 - 320 + 160 = \boxed{544}$.
DPatrick 2017-03-24 20:56:13
12. Circle $C_0$ has radius 1, and the point $A_0$ is a point on the circle. Circle $C_1$ has radius $r < 1$ and is internally tangent to $C_0$ at point $A_0$. Point $A_1$ lies on circle $C_1$ so that $A_1$ is located $90^\circ$ counterclockwise from $A_0$ on $C_1$. Circle $C_2$ has radius $r^2$ and is internally tangent to $C_1$ at point $A_1$. In this way a sequence of circle $C_1,C_2,C_3,\ldots$ and a sequence of points on the circles $A_1,A_2,A_3,\ldots$ are constructed, where circle $C_n$ has radius $r^n$ and is internally tangent to circle $C_{n-1}$ at point $A_{n-1}$, and point $A_n$ lies on $C_n$ $90^\circ$ counterclockwise from point $A_{n-1}$, as shown in the figure below. There is one point $B$ inside all of these circles. When $r = \frac{11}{60}$, the distance from the center of $C_0$ to $B$ is $\frac{m}{n}$, where $m$ and $n$ are relatively prime positive integers. Find $m+n$.
DPatrick 2017-03-24 20:56:17
DPatrick 2017-03-24 20:57:11
This is not nearly as hard a problem as it looks, once you read it carefully and decipher what's going on.
Mudkipswims42 2017-03-24 20:57:25
Well... let's get bashing!
DPatrick 2017-03-24 20:57:35
Yeah...we can set up a coordinate system so that the center of the original circle is the origin, and $A_0 = (1,0)$.
DPatrick 2017-03-24 20:57:47
In this coordinate system, what's point $A_1$?
DPatrick 2017-03-24 20:58:19
To figure this out, think about how we "move" from $A_0$ to $A_1$.
DPatrick 2017-03-24 20:58:50
And let's everything in terms of $r$ and not plug in the given fraction...that'll be a mess. Let's not use $\frac{11}{60}$ until we absolutely have to.
to_chicken 2017-03-24 20:58:56
$(1-r,r)$
Piazolla13 2017-03-24 20:58:56
(1-r,r)
DPatrick 2017-03-24 20:59:07
Right! To get from $A_0$ to $A_1$, we move left $r$ units and up $r$ units.
DPatrick 2017-03-24 20:59:20
(Because the circle we're moving along has radius $r$.)
DPatrick 2017-03-24 20:59:27
So $A_1 = (1-r,r)$.
DPatrick 2017-03-24 20:59:31
Now what's $A_2$?
Mudkipswims42 2017-03-24 20:59:57
$(1-r-r^2,~r-r^2)$?
GeronimoStilton 2017-03-24 20:59:57
$A_2 = (1-r-r^2, r-r^2)$
SomethingNeutral 2017-03-24 21:00:01
$A_2 = (1-r-r^2, r-r^2)
DPatrick 2017-03-24 21:00:04
To get from $A_1$ to $A_2$, we move left $r^2$ units and down $r^2$ units.
DPatrick 2017-03-24 21:00:08
So $A_2 = (1-r-r^2,r-r^2)$.
DPatrick 2017-03-24 21:00:18
What's $A_3$?
RedPhoenix 2017-03-24 21:00:44
(1-r-r^2+r^3, r-r^2-r^3)
DPatrick 2017-03-24 21:00:52
To get from $A_2$ to $A_3$, we move right $r^3$ units and down $r^3$ units.
DPatrick 2017-03-24 21:00:56
So $A_3 = (1-r-r^2+r^3,r-r^2-r^3)$.
DPatrick 2017-03-24 21:01:01
See the pattern?
DPatrick 2017-03-24 21:01:08
To get from $A_{n-1}$ to $A_n$, we move $r^n$ units left or right and $r^n$ units up or down.
DPatrick 2017-03-24 21:01:22
Furthermore, for both the vertical and horizontal directions, we do two moves in the same direction, then two moves in the opposite direction, etc.
DPatrick 2017-03-24 21:01:49
So $A_n = (1-r-r^2+r^3+r^4-r^5-r^6+r^7+\cdots\pm r^n,$

$r-r^2-r^3+r^4+r^5-r^6-r^7+\cdots\pm r^n)$.
DPatrick 2017-03-24 21:02:02
And how does $B$ relate to all this?
Mudkipswims42 2017-03-24 21:02:11
Infinite!
SomethingNeutral 2017-03-24 21:02:15
B is when n approaches infinity
DPatrick 2017-03-24 21:02:26
Right. Notice that $A_n$ is inside all of the previous circles $C_0,C_1,\ldots,C_{n-1}$.
DPatrick 2017-03-24 21:02:37
So if we take the "limit" point of the $A_n$'s as $n$ goes to $\infty$, we'll get a point that's inside all of the $C$'s.
DPatrick 2017-03-24 21:02:46
That is, $B$ is the point whose $x$-coordinate is the infinite series \[1-r-r^2+r^3+r^4-r^5-r^6+r^7+\cdots\] and whose $y$-coordinate is the infinite series \[r-r^2-r^3+r^4+r^5-r^6-r^7+\cdots\]where the terms are powers of $r$ and the signs change every other term.
DPatrick 2017-03-24 21:03:03
How do we sum these series?
carzland 2017-03-24 21:03:13
Geometric
GeronimoStilton 2017-03-24 21:03:13
Geometric Series!
SomethingNeutral 2017-03-24 21:03:13
they are geometric!!
DPatrick 2017-03-24 21:03:32
Really? At first glance, they don't look geometric to me, because of the changing signs.
DaAsianPotatoe 2017-03-24 21:03:49
each term has two terms
DPatrick 2017-03-24 21:04:08
Aha! The first one is really a geometric series if we group the terms in pairs: \[(1-r) - r^2(1-r) + r^4(1-r) - r^6(1-r) + \cdots.\]
DPatrick 2017-03-24 21:04:27
That is, it's the infinite geometric series with first term $1-r$ and common ratio $-r^2$.
DPatrick 2017-03-24 21:04:36
What does it sum to?
reaganchoi 2017-03-24 21:04:53
$\frac{1-r}{1-(-r^2)}$
Mudkipswims42 2017-03-24 21:04:53
$\dfrac{1-r}{1+r^2}$
tdeng 2017-03-24 21:04:53
$\frac{1-r}{1+r^2}$
GeronimoStilton 2017-03-24 21:04:53
$\frac{1-r}{1+r^2}$
DPatrick 2017-03-24 21:05:02
Right: first term $1-r$ and common ratio $-r^2$, so its sum is $\dfrac{1-r}{1+r^2}$.
DPatrick 2017-03-24 21:05:15
So that's the $x$-coordinate of $B$. What about the $y$-coordinate?
DPatrick 2017-03-24 21:05:31
(You might have to scroll up a little to see it.)
DPatrick 2017-03-24 21:05:44
I'll repost it: $B$ is the point whose $x$-coordinate is the infinite series \[1-r-r^2+r^3+r^4-r^5-r^6+r^7+\cdots\] and whose $y$-coordinate is the infinite series \[r-r^2-r^3+r^4+r^5-r^6-r^7+\cdots\]where the terms are powers of $r$ and the signs change every other term.
GeronimoStilton 2017-03-24 21:05:50
It's $\frac{r(1-r)}{1+r^2}$
reaganchoi 2017-03-24 21:05:50
$\frac{r(1-r)}{1-(-r^2)}$
DPatrick 2017-03-24 21:06:03
Indeed. Notice that the second series (the $y$-coordinate expression) is just $r$ times the first one.
DPatrick 2017-03-24 21:06:15
This means that $B = \left(\dfrac{1-r}{1+r^2},\dfrac{r(1-r)}{1+r^2}\right)$.
DPatrick 2017-03-24 21:06:22
And what is the distance from $B$ to the origin?
Metal_Bender19 2017-03-24 21:06:57
(1-r)/($\sqrt{1+r^2}$)
DPatrick 2017-03-24 21:07:17
Right -- notice there's a big constant $\dfrac{1-r}{1+r^2}$ in common.
DPatrick 2017-03-24 21:07:26
So it's just $\dfrac{1-r}{1+r^2}\sqrt{1+r^2} = \dfrac{1-r}{\sqrt{1+r^2}}$.
SomethingNeutral 2017-03-24 21:07:40
plug in 11/60 now
a1b2 2017-03-24 21:07:40
$\frac{11}{60}
DPatrick 2017-03-24 21:07:46
Right, we can't put it off any longer.
DPatrick 2017-03-24 21:07:51
Now we can plug in $r = \dfrac{11}{60}$ and see what we get.
DPatrick 2017-03-24 21:07:55
\[\frac{1-r}{\sqrt{1+r^2}} = \frac{\frac{49}{60}}{\sqrt{1+\frac{11^2}{60^2}}}.\]
DPatrick 2017-03-24 21:08:10
Multiply by 60/60 and this simplifies to $\dfrac{49}{\sqrt{60^2 + 11^2}}$.
letsgomath 2017-03-24 21:08:24
49/61
soojoong 2017-03-24 21:08:24
49/61!!!!!!!!!!!!
RedPhoenix 2017-03-24 21:08:24
the denominator is just 61
DPatrick 2017-03-24 21:08:31
If you know your Pythagorean Triples well, and in particular notice that 11-60-61 is a triple, you can quickly see that the denominator is just 61. So the distance we want is $\dfrac{49}{61}$.
DPatrick 2017-03-24 21:09:03
Or you can compute it...but given the way the problem is phrased, we know it that $60^2 + 11^2$ has to be a perfect square.
skiboy32 2017-03-24 21:09:11
so the answer is $49+61=\boxed{110}.$
DPatrick 2017-03-24 21:09:18
This fraction is in lowest terms, so our answer is $49 + 61 = \boxed{110}$.
DPatrick 2017-03-24 21:09:36
13. For each integer $n \ge 3$, let $f(n)$ be the number of 3-element subsets of the vertices of a regular $n$-gon that are the vertices of an isosceles triangle (including equilateral triangles). Find the sum of all values of $n$ such that $f(n+1) = f(n) + 78$.
DPatrick 2017-03-24 21:09:58
It'd be nice if there were a super-easy formula for $f(n)$ to plug into, but there's probably not. (The problem would be too easy for a #13 if there were.)
DPatrick 2017-03-24 21:10:04
Indeed, what sorts of things do we have to worry about?
Mudkipswims42 2017-03-24 21:10:41
well there are 4 cases
Mudkipswims42 2017-03-24 21:10:41
parity, and equilaterals
Tan 2017-03-24 21:10:41
divisible by 2 or 3
High 2017-03-24 21:10:41
when n is a multiple of 3
skiboy32 2017-03-24 21:10:41
if it's divisible by 3, because we then have equilateral triangles
RedPhoenix 2017-03-24 21:10:41
equilateral triangles, odd or even....
DPatrick 2017-03-24 21:10:52
Right. The calculation is slightly different for $n$ odd and $n$ even, and the calculation is slightly different if $n$ is a multiple of 3.
DPatrick 2017-03-24 21:11:12
Let's first look at when $n$ not a multiple of 3, since this is easier: there are no equilateral triangles, so every isosceles triangle has a unique base side.
DPatrick 2017-03-24 21:11:18
An example might help. Let's look at $n=7$:
DPatrick 2017-03-24 21:11:24
DPatrick 2017-03-24 21:11:32
If I draw any diagonal, can I make an isosceles triangle, and if so how many?
DPatrick 2017-03-24 21:11:51
More specifically, can I make an isosceles triangle with my diagonal as its base?
skiboy32 2017-03-24 21:12:08
yes, and 1
RedPhoenix 2017-03-24 21:12:08
yes, exactly 1
DPatrick 2017-03-24 21:12:13
DPatrick 2017-03-24 21:12:25
There's exactly one. I need to take the "midpoint" of the side of the blue line with an odd number of vertices, and use that as my third point:
DPatrick 2017-03-24 21:12:29
DPatrick 2017-03-24 21:12:46
I can't do it on the other side of the blue segment, since there's no "midpoint".
DPatrick 2017-03-24 21:12:55
Since $n-2$ is odd, only one side of the base will have an odd number of vertices, so we can only make one isosceles triangle with that base.
DPatrick 2017-03-24 21:13:06
And this works even if we pick an edge of the $n$-gon to be our base, since there are $n-2$ (an odd number) of vertices left.
DPatrick 2017-03-24 21:13:09
DPatrick 2017-03-24 21:13:26
So if $n$ is odd (and not a multiple of 3), how many isosceles triangles are there?
_--__--_ 2017-03-24 21:13:49
n(n-1)/2
DaAsianPotatoe 2017-03-24 21:13:49
nC2
fdas 2017-03-24 21:13:49
(n-1)/2
DPatrick 2017-03-24 21:13:56
Right: if $n$ is odd and not a multiple of 3, the number of isosceles triangles is just the number of pairs of vertices.
DPatrick 2017-03-24 21:14:02
This is $\dbinom{n}{2} = \dfrac{n(n-1)}{2}$.
DPatrick 2017-03-24 21:14:40
To recap: if $n$ is odd, not a multiple of 3, we can pick any diagonal or edge to be our base, and there will be one isosceles triangle on the "odd" side of that segment.
DPatrick 2017-03-24 21:14:46
What if $n$ is even? Let's take $n=8$ as our example:
DPatrick 2017-03-24 21:14:49
DPatrick 2017-03-24 21:14:56
Can every diagonal be a base of an isosceles triangle?
willmathxu 2017-03-24 21:15:09
no
vishwathganesan 2017-03-24 21:15:09
no
Metal_Bender19 2017-03-24 21:15:09
no
legolego 2017-03-24 21:15:09
no
DPatrick 2017-03-24 21:15:14
Why not?
SomethingNeutral 2017-03-24 21:15:35
if you take the side that splits into a trapezoid
skiboy32 2017-03-24 21:15:35
Try when you choose vertices 3 over, for instance
a1b2 2017-03-24 21:15:38
Split into $2$ parts with an even parity of points
DPatrick 2017-03-24 21:15:43
If the diagonal splits the rest of the vertices into two even groups (which is possible because $n-2$ is even), then we can't pick a "middle" vertex on either side to make an isosceles triangle:
DPatrick 2017-03-24 21:15:49
DPatrick 2017-03-24 21:16:04
The above blue diagonal is not the base of any isosceles triangle.
DPatrick 2017-03-24 21:16:14
But what if the diagonal splits the other $n-2$ vertices into two odd groups?
legolego 2017-03-24 21:16:33
two triangles
SomethingNeutral 2017-03-24 21:16:33
then 2 triangles
vishwathganesan 2017-03-24 21:16:33
then 2 triiangles
skiboy32 2017-03-24 21:16:33
Then, it can make two.
DPatrick 2017-03-24 21:16:43
If the diagonal splits the other $n-2$ vertices into two odd groups, then we get 2 isosceles triangles, one on each side:
DPatrick 2017-03-24 21:16:46
DPatrick 2017-03-24 21:16:52
So the number of triangles is $2 \cdot \text{(number of diagonals an even # of vertices apart)}$.
DPatrick 2017-03-24 21:17:00
How many of those diagonals are there?
DPatrick 2017-03-24 21:18:03
Indeed, in the octagon $n=8$ there are 12.
DPatrick 2017-03-24 21:18:08
What's the formula for general even $n$?
DPatrick 2017-03-24 21:18:58
Think of the vertices in two sets of $n/2$: the "even" vertices and the "odd" vertices,
DPatrick 2017-03-24 21:19:13
We can pick two of the evens and we're OK, or we can pick two of the odds and we're OK.
DPatrick 2017-03-24 21:19:28
So the number of such diagonals is $2\dbinom{n/2}{2}$.
DPatrick 2017-03-24 21:19:52
And remember that each of these diagonals gives us 2 isosceles triangles!
DPatrick 2017-03-24 21:19:59
So the number of isosceles triangles is $4\dbinom{n/2}{2} = 4\dfrac{\frac{n}{2}\cdot\frac{n-2}{2}}{2} = \dfrac{n(n-2)}{2}$.
DPatrick 2017-03-24 21:20:13
To recap so far: if $n$ is not a multiple of 3, then \[ f(n) = \left\{\begin{array}{ll} \dfrac{n(n-1)}{2} & \text{if $n$ is odd} \\ \dfrac{n(n-2)}{2} & \text{if $n$ is even}\end{array}\right.\]
DPatrick 2017-03-24 21:20:32
And now, what happens when $n$ is a multiple of 3?
skiboy32 2017-03-24 21:20:55
Then, the equilateral triangle make us overcount.
DPatrick 2017-03-24 21:21:11
Right. Our count above hinged on the fact that every isosceles triangle had a unique base.
DPatrick 2017-03-24 21:21:32
But when $n$ is a multiple of 3, we get some equilateral triangles by taking 3 evenly-spaced vertices of the $n$-gon.
DPatrick 2017-03-24 21:21:45
How many equilateral triangles are there?
SomethingNeutral 2017-03-24 21:22:09
n/3
legolego 2017-03-24 21:22:09
n/3
jonzli123 2017-03-24 21:22:09
n/3
tdeng 2017-03-24 21:22:09
n/3
DPatrick 2017-03-24 21:22:17
Right. The equilateral triangle can be rotated into $n/3$ different positions, so there are $n/3$ of them.
DPatrick 2017-03-24 21:22:26
How many times has each one been counted in our $f(n)$ formula above?
skiboy32 2017-03-24 21:22:37
3.
legolego 2017-03-24 21:22:37
three times
flamescarlet 2017-03-24 21:22:37
3
abishek99 2017-03-24 21:22:37
3
DPatrick 2017-03-24 21:22:41
3 times: once for each "base".
DPatrick 2017-03-24 21:22:49
So they've been triple-counted, which means they've been double-overcounted.
DPatrick 2017-03-24 21:23:07
So, how do we correct our $f(n)$ formula when $n$ is a multiple of 3?
vishwathganesan 2017-03-24 21:23:25
-2n/3
Diophantine33 2017-03-24 21:23:27
Subtract 2n/3
SomethingNeutral 2017-03-24 21:23:27
subtract $\dfrac{2n}3$
DPatrick 2017-03-24 21:23:47
Right. There are $\dfrac{n}{3}$ equilaterals, and they got doubly-overcounted. So we have to subtract $\dfrac{2n}{3}$ whenever $n$ is a multiple of 3.
DPatrick 2017-03-24 21:23:53
Thus, with a little abuse of notation, we can write: \[ f(n) = \left\{\begin{array}{ll} \dfrac{n(n-1)}{2} & \text{if $n$ is odd} \\ \dfrac{n(n-2)}{2} & \text{if $n$ is even}\end{array}\right\} - \left\{\begin{array}{ll} 0 & \text{if $n$ is not a multiple of 3} \\ \dfrac{2n}{3} & \text{if $n$ is a multiple of 3}\end{array}\right\}\]
DPatrick 2017-03-24 21:24:02
Ick. This seems like a lot to keep track of!
DPatrick 2017-03-24 21:24:19
What would make this simpler to work with?
azmath333 2017-03-24 21:24:26
now bash out mod 6
DPatrick 2017-03-24 21:24:32
Sure. We could look at the different residues mod 6 and get a formula for each one.
DPatrick 2017-03-24 21:24:40
For example, what's $f(6k)$?
DPatrick 2017-03-24 21:24:58
That's the "even, is a multiple of 3" case.
DPatrick 2017-03-24 21:25:19
So it works out to \[f(6k) = \dfrac{(6k)(6k-2)}{2} - \dfrac{2(6k)}{3}.\]
DPatrick 2017-03-24 21:25:38
This simplifies to $f(6k) = 18k^2 - 10k$.
DPatrick 2017-03-24 21:25:59
I'll run through the other calculations, since they're not that interested to spend time on.
DPatrick 2017-03-24 21:26:03
$f(6k+1) = \dfrac{(6k+1)(6k+1-1)}{2} = 18k^2 + 3k$.
DPatrick 2017-03-24 21:26:09
$f(6k+2) = \dfrac{(6k+2)(6k+2-2)}{2} = 18k^2 + 6k$.
DPatrick 2017-03-24 21:26:13
$f(6k+3) = \dfrac{(6k+3)(6k+3-1)}{2} - \dfrac{2(6k+3)}{3} = 18k^2 + 11k + 1$.
DPatrick 2017-03-24 21:26:17
$f(6k+4) = \dfrac{(6k+4)(6k+4-2)}{2} = 18k^2 + 18k + 4$.
DPatrick 2017-03-24 21:26:20
$f(6k+5) = \dfrac{(6k+5)(6k+5-1)}{1} = 18k^2 + 27k + 10$.
DPatrick 2017-03-24 21:26:30
That's a lot of data. Now what?
DPatrick 2017-03-24 21:27:05
By now we may have forgotten the original problem, so a glance back might help...
GeronimoStilton 2017-03-24 21:27:08
We look at the difference between these.
carzland 2017-03-24 21:27:10
Find the one that is 78 apart.
DPatrick 2017-03-24 21:27:18
Right. We care about when $f(n+1) - f(n) = 78$. So let's define $g(n) = f(n+1)-f(n)$ and compute it for our mod 6 residues.
DPatrick 2017-03-24 21:27:36
\begin{align*}

g(6k) &= f(6k+1)-f(6k) = 13k \\

g(6k+1) &= f(6k+2)-f(6k+1) = 3k \\

g(6k+2) &= f(6k+3)-f(6k+2) = 5k+1 \\

g(6k+3) &= f(6k+4)-f(6k+3) = 7k+3 \\

g(6k+4) &= f(6k+5)-f(6k+4) = 9k+6 \\

g(6k+5) &= f(6k+6)-f(6k+5) = -k-2

\end{align*}
DPatrick 2017-03-24 21:27:46
(Again, the calculations are not that interesting.)
DPatrick 2017-03-24 21:27:55
Which of these $g$'s could possibly equal 78?
QuestForKnowledge 2017-03-24 21:28:24
the 5th one or the first
carzland 2017-03-24 21:28:24
The second.
DPatrick 2017-03-24 21:28:29
We just have to run through them.
DPatrick 2017-03-24 21:28:34
$g(6k) = 13k = 78$ is true when $k=6$. So $n = 6k = 36$ is a solution.



$g(6k+1) = 3k = 78$ is true when $k=26$. So $n = 6k+1 = 6(26)+1 = 157$ is a solution.



$g(6k+2) = 5k+1 = 78$ simplifies to $5k = 77$. This has no solution.



$g(6k+3) = 7k+3 = 78$ simplifies to $7k = 75$. This has no solution.



$g(6k+4) = 9k+6 = 78$ simplifies to $9k = 72$, so $k=8$ works. So $n = 6k+4 = 6(8)+4 = 52$ is a solution.



$g(6k+5) = -k-2 = 78$ has no solution (the left side is negative!).
DPatrick 2017-03-24 21:28:58
So the solutions are $n=36$, $n=157$, and $n=52$.
DPatrick 2017-03-24 21:29:11
Our final answer is $36 + 157 + 52 = \boxed{245}$.
DPatrick 2017-03-24 21:29:45
This problem was mainly a lot of bookkeeping.
GeronimoStilton 2017-03-24 21:30:04
What do you mean by 'bookkeeping'?
DPatrick 2017-03-24 21:30:12
Just a lot of data to accurately collect.
DPatrick 2017-03-24 21:30:20
Two to go!
DPatrick 2017-03-24 21:30:23
14. A $10 \times 10 \times 10$ grid of points consists of all points in space of the form $(i,j,k)$, where $i$, $j$, and $k$ are integers between 1 and 10, inclusive. Find the number of different lines that contain exactly 8 of these points.
DPatrick 2017-03-24 21:30:53
There are lots of ways to approach this problem. Most involve (again) careful bookkeeping.
SomethingNeutral 2017-03-24 21:31:08
Diagram
DPatrick 2017-03-24 21:31:31
We can try to draw a diagram. I'm not going to try to draw a cube of 100 points, though.
DPatrick 2017-03-24 21:31:42
In fact, I like 2-D better than 3-D, so I'm going to project my line down to the $xy$-plane (that is, set all the $z$-coordinates to 0):
DPatrick 2017-03-24 21:31:45
DPatrick 2017-03-24 21:32:24
If we have a line that works, and we project it down to the $xy$-plane, we still have 8 points on a line segment in this plane.
DPatrick 2017-03-24 21:33:01
So we can try to count the 8-point segments in this 10x10 grid, and count how many different ways we could "unproject" it up into our cube.
DPatrick 2017-03-24 21:33:18
Could I have an 8-point segment in my 10x10 grid that doesn't hit the boundary, like so?
DPatrick 2017-03-24 21:33:21
legolego 2017-03-24 21:33:47
no
GeronimoStilton 2017-03-24 21:33:47
No.
DPatrick 2017-03-24 21:33:50
Why not?
abishek99 2017-03-24 21:34:24
contains more than 8 points
DPatrick 2017-03-24 21:34:58
Right. When I lift this up into the cube, I'm guaranteed to have enough room at one end or the other to extend to a 9th point!
DPatrick 2017-03-24 21:35:13
Maybe this is easier to see if we compare to this line segment:
DPatrick 2017-03-24 21:35:16
DPatrick 2017-03-24 21:35:30
Can I lift this segment in the $xy$-plane up to the cube so that it only contains 8 points of the cube?
SomethingNeutral 2017-03-24 21:35:56
Yes
62861 2017-03-24 21:36:03
yes, if it contains lattice points from $z = 1, 2, \dots, 8$?
DPatrick 2017-03-24 21:36:11
Right!
DPatrick 2017-03-24 21:36:33
We're OK, provided that the non-boundary end of the segment (the point $(8,5)$ in my example) hits either $z=1$ or $z=10$, so that I can't extend it any further and pick up a 9th point.
DPatrick 2017-03-24 21:36:50
For example, with the red segment shown running from $(1,5)$ to $(8,5)$, I could have the line in the cube passing through \[(1,5,8),(2,5,7),(3,5,6),\ldots,(8,5,1)\] or the line in the cube passing through \[(1,5,3),(2,5,4),(3,5,5),\ldots,(8,5,10)\] and we don't get a ninth point on that line in our cube.
DPatrick 2017-03-24 21:37:31
(Because there's no room in the $z$ direction to the ninth point at $(9,5)$ in the plane.)
DPatrick 2017-03-24 21:38:08
So each 8-point segment in the $10 \times 10$ grid that has one endpoint on the boundary gives us 2 valid lines.
DPatrick 2017-03-24 21:38:21
How many such segments are there?
SomethingNeutral 2017-03-24 21:38:51
40
DPatrick 2017-03-24 21:38:55
Each row has 2, one at each end, so that's 20.
DPatrick 2017-03-24 21:39:00
Each column has 2, one at each end, so that's another 20.
DPatrick 2017-03-24 21:39:09
But there are some diagonal ones too!
DPatrick 2017-03-24 21:39:27
The three middle southwest-to-northeast diagonals each give us 2, one at each end, like so (I'm showing you the one at the southwest end below), so that's another 6:
DPatrick 2017-03-24 21:39:32
DPatrick 2017-03-24 21:39:45
And the three middle northwest-to-southeast diagonals each gives us 2, one at each end, so that's another 6.
DPatrick 2017-03-24 21:40:01
So we have a total of $20+20+6+6 = 52$ 8-point segments in our grid that hits the boundary at one end of the grid.
DPatrick 2017-03-24 21:40:07
And each of these gave us 2 lines in the cube, so that's a total of $2 \cdot 52 = 104$ lines so far.
DPatrick 2017-03-24 21:40:29
And finally, what if we start with an 8-point segment that hits the boundary of the $xy$-plane at both ends?
DPatrick 2017-03-24 21:40:32
DPatrick 2017-03-24 21:40:45
In how many ways can each of these be lifted up to the cube to make a valid line?
vishwathganesan 2017-03-24 21:41:07
a lot
DPatrick 2017-03-24 21:41:31
Indeed, now there's very little restriction on the $z$-coordinates. We can't get a 9th point even in the $xy$-plane, so we don't have to worry about $z$ so much.
GeronimoStilton 2017-03-24 21:41:36
$16$
DPatrick 2017-03-24 21:41:56
Right: there are 16 ways to lift each segment up to the cube, as follows.
DPatrick 2017-03-24 21:42:03
$z$ could be constant. That's 10 possibilities.
DPatrick 2017-03-24 21:42:08
$z$ could start at 1, 2, or 3 and increase as we go left-to-right. That's 3 possibilities.
DPatrick 2017-03-24 21:42:16
$z$ could start at 8, 9, or 10 and decrease as we go left-to-right. That's 3 possibilities.
DPatrick 2017-03-24 21:42:26
So each 8-point segment in the picture above gives us $10+3+3=16$ lines in the cube.
vishwathganesan 2017-03-24 21:42:52
so sixty4 total
DPatrick 2017-03-24 21:43:01
There were 4 such segments in the planar grid, so that's a total of $4 \cdot 16 = 64$ lines in the cube.
DPatrick 2017-03-24 21:43:05
Adding this to the $104$ lines we found earlier gives us a grand total of $64 + 104 = \boxed{168}$ total lines.
DPatrick 2017-03-24 21:43:29
I thought that this problem, by far, was the hardest problem on the contest.
DPatrick 2017-03-24 21:43:53
#15 by comparison is a lot shorter.
DPatrick 2017-03-24 21:44:03
15. Tetrahedron $ABCD$ has $AD=BC=28$, $AC=BD=44$, and $AB=CD=52$. For any point $X$ in space, define $f(X) = AX + BX + CX + DX$. The least possible value of $f(X)$ can be expressed as $m\sqrt{n}$, where $m$ and $n$ are positive integers, and $n$ is not divisible by the square of any prime. Find $m+n$.
DPatrick 2017-03-24 21:44:31
Of course, we notice that this is a tetrahedron whose opposite sides are equal.
DPatrick 2017-03-24 21:44:36
So there's probably some symmetry that we can use somewhere.
DPatrick 2017-03-24 21:45:02
Is there a clever way to embed this tetrahedron in 3-D Cartesian space?
flamescarlet 2017-03-24 21:45:27
as a rectangle?
carzland 2017-03-24 21:45:30
The middle is the origin.
agbdmrbirdyface 2017-03-24 21:45:32
symmetric about the origin?
DPatrick 2017-03-24 21:45:42
How about as every other vertex of a rectangular box?
DPatrick 2017-03-24 21:45:55
Specifically, $A,B,C,D$ are four of the eight corners of a rectangular box with face diagonals of 28, 44, and 52, and with sides $2r$, $2s$, and $2t$, and center at the origin.
DPatrick 2017-03-24 21:45:59
DPatrick 2017-03-24 21:46:09
In coordinates, we can set $A = (r,s,t)$, $B = (-r,-s,t)$, $C = (-r,s,-t)$, and $D = (r,-s,-t)$ for some positive values of $r,s,t$.
DPatrick 2017-03-24 21:46:24
So now our pairs of equal opposite sides are the face diagonals of our box.
agbdmrbirdyface 2017-03-24 21:46:41
and now we use distance formula?
DPatrick 2017-03-24 21:46:53
Yeah: the computation part of the solution is now pretty straightforward.
GeronimoStilton 2017-03-24 21:46:58
We want $4\sqrt{r^2 + s^2 + t^2}$!
DPatrick 2017-03-24 21:47:09
Right, and we have \begin{align*}

28 = AD = BC &= 2\sqrt{s^2+t^2}, \\

44 = AC = BD &= 2\sqrt{r^2+t^2}, \\

52 = AB = CD &= 2\sqrt{r^2+s^2}.

\end{align*}
DPatrick 2017-03-24 21:47:20
This simplifies to the system \begin{align*}

196 = 14^2 &= s^2 + t^2, \\

484 = 22^2 &= r^2 + t^2, \\

676 = 26^2 &= r^2 + s^2.

\end{align*}
DPatrick 2017-03-24 21:47:35
Now, it's a pretty good guess that the origin $(0,0,0)$ is the point $X$ that we want. This point is the center of the tetrahedron.
GeronimoStilton 2017-03-24 21:47:53
Add them and then divide by 2
vishwathganesan 2017-03-24 21:47:53
add everything together
agbdmrbirdyface 2017-03-24 21:47:53
add them up, divide by 2, square root, multiply by 4
DPatrick 2017-03-24 21:48:22
So we see that $f((0,0,0)) = 4\sqrt{r^2+s^2+t^2}$, and we can get this quantity by summing our system of equations.
DPatrick 2017-03-24 21:48:28
Summing our system gives $196 + 484 + 676 = 1356 = 2(r^2+s^2+t^2)$, so $r^2 + s^2 + t^2 = \dfrac{1356}{2} = 678$.
DPatrick 2017-03-24 21:48:40
And this makes $f((0,0,0)) = 4\sqrt{678}$.
DPatrick 2017-03-24 21:48:48
This doesn't simplify further, so it suggests that $4 + 678 = \boxed{682}$ is the answer.
62861 2017-03-24 21:48:53
why is $X$ the origin
DPatrick 2017-03-24 21:48:58
Aha...there's the rub.
DPatrick 2017-03-24 21:49:06
This isn't rigorous yet, because we guessed that $(0,0,0)$ is the point we want. Can we prove that?
DPatrick 2017-03-24 21:49:23
Let's go back to the picture:
DPatrick 2017-03-24 21:49:27
DPatrick 2017-03-24 21:49:38
Suppose $X$ is an arbitrary point. How can we show that $f((0,0,0)) \le f(X)$?
flamescarlet 2017-03-24 21:49:54
triangle ineq?
DPatrick 2017-03-24 21:50:04
It's basically a Triangle Inequality argument.
DPatrick 2017-03-24 21:50:12
We thought a lot about this here, and we couldn't find a good way to show it "all at once" using symmetry. Most of the solutions we came up with (and most of the solutions I've seen posted) first show that $X$ can be moved to one of the axes to decrease $f(X)$. And then, you do that twice (with two different axes) to move it to the origin.
DPatrick 2017-03-24 21:51:01
In particular, let's show that we can move $X$ "laterally" to the $z$-axis to decrease $f$. Suppose $X$ is an arbitrary point, and $Y$ is the point in the same $xy$-plane cross-section of $X$ that's on the $z$-axis. We'll try to show that $f(Y) \le f(X)$.
DPatrick 2017-03-24 21:51:08
Here's a "top-down" picture of what's going on, looking straight down from above:
DPatrick 2017-03-24 21:51:12
DPatrick 2017-03-24 21:51:22
Remember that these points are at different "heights": $A$ and $B$ are "on top" in the $z=t$ plane, $C$ and $D$ are "on the bottom" in the $z=-t$ plane, and $X$ and $Y$ are in some horizontal plane somewhere in the middle.
DPatrick 2017-03-24 21:51:32
We want to show that $f(Y) \le f(X)$.
DPatrick 2017-03-24 21:51:56
One idea is to reflect $X$ in its plane across the origin, creating a new point $X'$:
DPatrick 2017-03-24 21:52:00
DPatrick 2017-03-24 21:52:05
One nice thing that happens is that $Y$ is the midpoint of $\overline{XX'}$.
DPatrick 2017-03-24 21:52:09
But we also see that $f(X') = f(X)$...why?
GeronimoStilton 2017-03-24 21:52:29
Symmetry.
SomethingNeutral 2017-03-24 21:52:29
symmetry
flamescarlet 2017-03-24 21:52:35
A and B at the same height, everything's rotated 180
DPatrick 2017-03-24 21:52:42
Exactly. This same reflection across the $z$-axis moves $A$ to $B$ and vice versa, and moves $C$ to $D$ and vice versa. In particular, $AX = BX'$ and $BX = AX'$, and the same for $C$ and $D$.
DPatrick 2017-03-24 21:53:00
So now, if we can show that $AX + AX' \ge 2(AY)$ (and the same for the other points), we'd be done, because then when we sum over all four points, we'd have $f(X) + f(X') \ge 2f(Y)$, or $f(X) \ge f(Y)$.
DPatrick 2017-03-24 21:53:19
So let's try to show that $AX + AX' \ge 2(AY)$. But this is much simpler since all these points lie in the same plane:
DPatrick 2017-03-24 21:53:22
DPatrick 2017-03-24 21:53:29
This is a general statement about triangles: if $AXX'$ is a triangle and $Y$ is the midpoint of $\overline{XX'}$, then $AX + AX' \ge 2(AY)$. Why is this true?
FieryEntei 2017-03-24 21:53:47
triangle inequality
mssmath 2017-03-24 21:53:47
Create a parallelogram
DPatrick 2017-03-24 21:53:52
Sure. Just reflect the triangle across $\overline{XX'}$ to make a parallelogram to point $Z$:
DPatrick 2017-03-24 21:53:55
DPatrick 2017-03-24 21:54:18
"Rotate" is probably a better word than "reflect" in my previous sentence.
DPatrick 2017-03-24 21:54:32
So now $AX' = ZX$ and $AY = ZY$, so $AX + AX' = AX + XZ$, and $2AY = AZ$.
DPatrick 2017-03-24 21:54:43
Since $AX + XZ \ge AZ$ by the Triangle Inequality, we have $AX + AX' \ge 2AY$, just like we wanted.
DPatrick 2017-03-24 21:54:58
To summarize: we've shown that we can take any point $X$ in our box and make $f(X)$ smaller by moving $X$ over to the $z$-axis.
DPatrick 2017-03-24 21:55:05
So doing this twice in a row (once with the $z$-axis and once with a different axis) shows that we can always make $f(X)$ smaller by moving it to the origin.
DPatrick 2017-03-24 21:55:13
Therefore, $f((0,0,0)) \le f(X)$ for any $X$, and indeed the origin is the best point.
DPatrick 2017-03-24 21:55:21
And thus, $f((0,0,0)) = 4\sqrt{678}$ is the least possible value of $f(X)$, and our final answer is $4 + 678 = \boxed{682}$.
DPatrick 2017-03-24 21:55:58
I thought this was easier than #14, AIME-wise, because you can "guess" that the center point is the best point without having to prove it. And if you put the tetrahedron in a box, the computation is pretty straightforward.
DPatrick 2017-03-24 21:56:15
#14 is just a nasty 3-D visualization and counting problem where you have to be really really careful.
GeronimoStilton 2017-03-24 21:56:21
We did all 15 problems in less than 3:00 hours!
DPatrick 2017-03-24 21:56:28
Success! That's it for the 2017 AIME II!
DPatrick 2017-03-24 21:56:37
Thanks for coming. Have a great weekend!

Copyright © 2024 AoPS Incorporated. This page is copyrighted material. You can view and print this page for your own use, but you cannot share the contents of this file with others.